Google Groups no longer supports new Usenet posts or subscriptions. Historical content remains viewable.
Dismiss

FAQ LaTeX francaise V2.12 part 1

2 views
Skip to first unread message

Marie-Paule Kluth

unread,
Feb 27, 1998, 3:00:00 AM2/27/98
to

Archive-name: fr/faq-latex-francaise/part1
Author: Marie-Pa...@aar.alcatel-alsthom.fr
Posting-Frequency: mensuel
Version: 2.12

Newsgroups: fr.comp.text.tex, fr.usenet.reponses, news.answers
Ftp-Archive-Name:
ftp://ftp.univ-lyon1.fr/pub/faq/by-name/fr/faq-latex-francaise

================================================================
---- ATTENTION cette FAQ est figee pour le moment ----
un grand merci a Benjamin Bayart qui s'est propose
pour continuer a la diffuser.
================================================================

fr.comp.text.tex est un forum français de discussion concernant
TeX et LaTeX. comp.text.tex est son équivalent anglophone. Cette
FAQ (recueil des questions les plus fréquentes) est
essentiellement composée à partir de messages postés dans ces
forums et de mon expérience personnelle.

Je tiens à souligner que cette FAQ LaTeX ne prétend pas être
correcte sur tous les points qu'elle énonce (j'essaie toutefois
de tester un maximum de réponses mais je ne dispose pas
toujours de tous les packages), ni même complète. C'est
pourquoi, je vous invite tous à y contribuer par vos remarques.
En outre, des exemples concis susceptibles d'illustrer l'une ou
l'autre des réponses seront les bienvenus.

Concernant l'évolution de LaTeX2.09 en LaTeX2e, sachant que
certains sites utilisent encore l'ancienne version (notamment
pour travailler sur d'anciens documents utilisant des styles ou
macros non compatibles avec LaTeX2e), cette FAQ présente encore
des aides pour LaTeX2.09. Cependant, il est entendu que ces
dernières seront amenées à disparaître.


################################################################

FAQ LaTeX version 2.12 (28/02/98)

################################################################
(première version : FAQ LaTeX version 1.0 (30/04/96))
(c) Copyright 1997 Marie-Paule Kluth
Tous droits réservés.

Note (destinée aux puristes) :
Afin de ne pas tomber dans l'excès qui consiste à traduire
toutes les expressions anglo-saxonnes utilisées dans LaTeX, en
français, on parlera, entre autres, dans ce qui suit de package
et non d'extension, de viewer, du WEB, etc.

Suite à de nombreuses demandes, les accents ont été ajoutés. Le
programme `recode' de GNU est capable d'en générer une version
sans les accents (recode latin1:flat faq_latex). Ceux qui ne
possèdent pas un tel programme peuvent me demander une version
sans accent par e-mail.

................................................................
Pour toutes remarques, commentaires ou ajouts, contactez moi
par e-mail : Marie-Pa...@aar.alcatel-alsthom.fr
................................................................

Si vous désirez, distribuer ce document par FTP ou sur le WEB,
ou placer un pointeur vers ce dernier, merci de m'en informer
par e-mail et de me communiquer l'adresse correspondante.

Redistribution for profit, or in altered content/format
prohibited without permission of the author.
Redistribution via printed book or CDROM expressly
prohibited without consent of the author. Any other
redistribution must include this copyright notice and
attribution.

Bonne lecture.

================================================================
THÈMES ABORDES :
================================================================

[1] PRÉSENTATION
----------------
# 1.1 # Quelle est l'histoire de (La)TeX ?
# 1.2 # Quels sont les principes de base de TeX ?
# 1.3 # Quels sont les principes de base de LaTeX ?
# 1.4 # Qu'est ce que le préambule du fichier source ?
# 1.5 # Comment faire ses premiers pas ?
# 1.6 # Comment sont gérées les options de package ?
# 1.7 # Quelle est la structure d'une page LaTeX ?
# 1.8 # Quelles sont les commandes de compilation LaTeX ?
# 1.9 # Quels sont les fichiers utilisés par LaTeX ?
# 1.10 # A quoi correspondent les messages Overfull ?

[2] DOCUMENTATION
-----------------
# 2.1 # Que puis je lire sur TeX ?
# 2.2 # Que puis je lire sur LaTeX2.09 ?
# 2.3 # Que puis je lire sur LaTeX2e ?
# 2.4 # Que puis je lire sur AMS-LaTeX ?
# 2.5 # Que puis-je lire sur la typographie ?
# 2.6 # Où trouver une bibliographie complète ?
# 2.7 # Que puis-je lire sur les fontes ?

[3] SITES WEB
-------------
# 3.1 # Où trouver des infos sur le WEB ?
# 3.2 # Où trouver cette FAQ ?
# 3.3 # Existe t'il des listes de discussion francophones ?
# 3.4 # Où trouver d'autres FAQ LaTeX ?

[4] SOURCES TEX ET LATEX
------------------------
# 4.1 # Où trouver les sources pour Unix ?
# 4.2 # Où trouver les sources pour VMS ?
# 4.3 # Où trouver les sources pour DOS, OS/2, Windows3.x/95/NT ?
# 4.4 # Où trouver les sources pour MacOS ?
# 4.5 # Où trouver d'autres sources ?

[5] ÉDITER LATEX
----------------
# 5.1 # Quel éditeur de texte puis je utiliser ?
# 5.2 # Qu'est ce que Lollipop ?

[6] GESTION DE LA MISE EN PAGE
------------------------------
# 6.1 # Comment modifier l'interligne d'un document ?
# 6.2 # Comment gérer un document recto-verso ?
# 6.3 # Comment modifier le style des titres ?
# 6.4 # Comment obtenir un document multicolonnes ?
# 6.5 # Comment composer une brochure ?
# 6.6 # Comment définir un format de document ?
# 6.7 # Comment redéfinir les marges d'un document ?
# 6.8 # Comment changer l'orientation d'un document ?
# 6.9 # Comment justifier verticalement un paragraphe ?
# 6.10 # Comment modifier la commande \caption ?
# 6.11 # Comment modifier un changement de page ?
# 6.12 # Comment obtenir des colonnes parallèles ?
# 6.13 # Comment définir l'espace de début de paragraphe ?
# 6.14 # Comment supprimer la date sur une page de titre ?
# 6.15 # Comment mettre en valeur la première lettre d'un
chapitre ?
# 6.16 # Comment préserver les tabulations en mode verbatim ?
# 6.17 # Comment modifier les entêtes de chapitre ?
# 6.18 # Comment définir des tabulations ?
# 6.19 # Comment obtenir des lettres accentuées dans tabbing ?
# 6.20 # Comment encadrer du texte ?
# 6.21 # Comment gérer des URL www ?
# 6.22 # Comment mettre en page des exercices dont les
solutions sont reportées dans un autre paragraphe ?
# 6.23 # Comment positionner un objet dans une page ?
# 6.24 # Comment lier le placement des flottants aux sections ?
# 6.25 # Comment griser le fond (background) d'un paragraphe ?
# 6.26 # Comment modifier l'espace inter-colonnes ?
# 6.27 # Comment modifier les environnements de liste ?
# 6.28 # Comment souligner plusieurs lignes ?
# 6.29 # Comment réaliser des onglets ?
# 6.30 # Comment réaliser des QCM ?
# 6.31 # Comment modifier l'orientation des flottants ?
# 6.32 # Comment faire référence plusieurs fois à la même note
de bas de page ?
# 6.33 # Comment éviter les orphelins en début ou fin de page ?
# 6.34 # Comment définir de nouveaux flottants ?
# 6.35 # Comment utiliser la commande \caption hors d'un
environnement flottant ?
# 6.36 # Comment encadrer un objet flottant ?
# 6.37 # Comment changer l'orientation d'un caption ?
# 6.38 # Comment mettre en page un programme ?
# 6.39 # Comment obtenir un espace insécable ?
# 6.40 # Comment insérer une page blanche ?
# 6.41 # Comment supprimer l'indentation des paragraphes ?
# 6.42 # Comment modifier l'espacement entre caractères ?
# 6.43 # Comment supprimer certaines coupures de mots ?
# 6.44 # Comment mettre en forme des équations chimiques ?
# 6.45 # Comment mettre en page un calendrier ?
# 6.46 # Comment forcer un caption sur plusieurs lignes ?
# 6.47 # Comment générer des barres de modification dans un
document ?
# 6.48 # Comment encadrer du texte verbatim ?
# 6.49 # Comment écrire du texte en forme de losange ou autre ?
# 6.50 # Comment isoler les flottants par un trait horizontal ?
# 6.51 # Comment insérer un espace vertical dans une page ?
# 6.52 # Comment insérer un espace horizontal dans un texte ?
# 6.53 # Comment définir un style de paragraphe ?
# 6.54 # A quoi sert la commande \stretch ?
# 6.55 # Comment justifier un paragraphe à gauche ou à droite ?
# 6.56 # Comment mettre en page des citations ?
# 6.57 # Comment insérer un texte sans que LaTeX le mette en
forme ?
# 6.58 # Comment visualiser des espaces ?
# 6.59 # Comment insérer du code LaTeX dans un document LaTeX ?
# 6.60 # Comment écrire du texte en couleurs ?
# 6.61 # Comment construire un arbre ?
# 6.62 # Comment mettre en page un poème ?
# 6.63 # Comment aligner des paragraphes ?
# 6.64 # Comment construire une liste d'éléments ?
# 6.65 # Comment faire une note dans une marge ?
# 6.66 # Comment faire une page de garde ?
# 6.67 # Comment utiliser \thanks dans chaque titre de chapitre ?
# 6.68 # Comment mettre un résumé et un abstract dans un
document ?
# 6.69 # Comment définir de nouvelles couleurs ?
# 6.70 # Comment placer des figures face à face en recto-verso ?
# 6.71 # Quelles sont les différences entre a4paper et
letterpaper ?
# 6.72 # Comment couper une chaîne de caractères ?
# 6.73 # Comment modifier l'espace entre une figure et sa
légende ?

[7] GESTION DES TABLEAUX
------------------------
# 7.1 # Comment faire un tableau ?
# 7.2 # Comment gérer les tableaux de plus d'une page ?
# 7.3 # Comment modifier l'orientation d'un tableau ?
# 7.4 # Comment nommer un tableau ?
# 7.5 # Comment modifier l'épaisseur des lignes d'un tableau ?
# 7.6 # Comment griser des cellules d'un tableau ?
# 7.7 # Comment changer la fonte d'une colonne ?
# 7.8 # Comment créer des notes de bas de page dans un tableau ?
# 7.9 # Comment écrire un texte sur plusieurs colonnes ?
# 7.10 # Comment passer certaines cellules en reverse vidéo ?
# 7.11 # Comment fixer la largeur d'une colonne ?
# 7.12 # Comment écrire un texte sur plusieurs lignes ?
# 7.13 # Comment diviser une cellule par une diagonale ?
# 7.14 # Comment définir une colonne en mode mathématique dans
un tableau ?
# 7.15 # Comment modifier le nombre de tableaux par page ?
# 7.16 # Comment définir un séparateur de colonne ?
# 7.17 # Comment obtenir des lignes partielles dans un tableau ?
# 7.18 # Comment éviter que du texte de grande taille atteigne
le cadre des cellules ?
# 7.19 # Comment fixer la largeur d'un tableau ?
# 7.20 # Comment tracer des traits discontinus ?
# 7.21 # Comment fixer la taille et justifier une colonne ?
# 7.22 # Comment faire une liste de tableaux ?

[8] GESTION DES FIGURES
-----------------------
# 8.1 # Comment inclure une figure ?
# 8.2 # Comment nommer une figure ?
# 8.3 # Comment placer des figures côte à côte ?
# 8.4 # Comment modifier le nombre de figures par page ?
# 8.5 # Comment superposer du texte sur des figures ?
# 8.6 # Comment réaliser des captures d'écran ?
# 8.7 # Comment tracer une courbe ?
# 8.8 # Comment est géré le positionnement des figures ?
# 8.9 # Comment placer une légende à côté d'une figure ?
# 8.10 # Comment insérer des figures dans multicol ?
# 8.11 # Comment faire apparaître toutes les figures en fin de
document ?
# 8.12 # Comment insérer des images Mathematica ?
# 8.13 # Comment modifier la taille d'une bounding box ?
# 8.14 # Comment obtenir une figure avec un titre non numéroté ?
# 8.15 # Comment redéfinir le style de caption ?
# 8.16 # Comment fondre une image dans du texte ?
# 8.17 # Comment réaliser des diagrammes en bâtons ?
# 8.18 # Comment faire un organigramme ?
# 8.19 # Comment centrer une figure très large ?
# 8.20 # Comment passer de ps à eps ?
# 8.21 # Comment changer l'orientation d'une figure ?
# 8.22 # Comment gérer des sous-figures sur plusieurs pages ?
# 8.23 # Comment générer une liste des figures d'un document ?
# 8.24 # Comment faire une figure sous LaTeX ?
# 8.25 # Comment mettre un commentaire à côté d'une figure ?

[9] INCLUSION DE FICHIERS
-------------------------
# 9.1 # Comment inclure des fichiers en mode verbatim ?
# 9.2 # Comment gérer un document par parties ?
# 9.3 # Comment isoler une partie d'un fichier ps ou dvi ?
# 9.4 # Comment inclure un fichier PICT ?
# 9.5 # Comment spécifier un chemin pour les fichiers à
inclure ?

[10] HAUTS ET BAS DE PAGES
--------------------------
# 10.1 # Comment définir les hauts et bas de page ?
# 10.2 # Comment obtenir une note de bas de page ?
# 10.3 # Comment mettre les notes de bas de page en fin de
document ?
# 10.4 # Comment réduire les rappels de titres dans un haut ou
bas de page ?
# 10.5 # Comment référencer une note de bas de page ?
# 10.6 # Comment supprimer la numérotation des pages ?
# 10.7 # Comment numéroter les pages par rapport à la dernière ?
# 10.8 # Comment supprimer le trait de séparation des notes de
bas de page ?
# 10.9 # Comment modifier la numérotation des pages ?
# 10.10 # Comment supprimer les entêtes et bas de page de pages
vierges ?
# 10.11 # Comment gérer des en-têtes avec des environnements
verbatim multi-pages ?
# 10.12 # Comment utiliser \footnote dans un titre ?
# 10.13 # Comment placer les notes de bas de page les unes à
côté des autres ?
# 10.14 # Comment réinitialiser le compteur de note de bas de
page à chaque page ?
# 10.15 # Comment modifier le style des notes de bas de page ?
# 10.16 # Comment utiliser le mode verbatim dans une note de bas
de page ?
# 10.17 # Comment éviter qu'une note de bas de page s'étale sur
plusieurs pages ?

[11] LATEX, LE FRANÇAIS ET LES AUTRES LANGUES
---------------------------------------------
# 11.1 # Comment franciser un document LaTeX ?
# 11.2 # Comment corriger les coupures de mots accentués ?
# 11.3 # Comment utiliser les lettres accentuées ?
# 11.4 # Comment composer du texte en grec moderne ou classique ?
# 11.5 # Comment composer du texte en hébreux moderne ou
classique ?
# 11.6 # Comment composer du texte en arabe
# 11.7 # Comment définir les césures de groupes de mots
conjugués avec un trait d'union ?

[12] MATHÉMATIQUES
------------------
# 12.1 # Comment passer en mode mathématique ?
# 12.2 # Qu'est que AMS-LaTeX ?
# 12.3 # Comment écrire les symboles d'ensembles ?
# 12.4 # Comment numéroter les équations ?
# 12.5 # Comment aligner des équations ?
# 12.6 # Comment générer des vecteurs ?
# 12.7 # Comment écrire du texte en mode mathématique ?
# 12.8 # Comment ajuster la taille de délimiteurs ?
# 12.9 # Comment changer de fonte en mode mathématique ?
# 12.10 # Comment obtenir le L de la transformée de Laplace ?
# 12.11 # Comment réaliser un tableau en mode mathématique ?
# 12.12 # Comment obtenir d'autres symboles mathématiques ?
# 12.13 # Comment définir une fonction ?
# 12.14 # Comment définir une matrice ?
# 12.15 # Comment encadrer des formules ?
# 12.16 # Comment ajuster la longueur d'une flèche par rapport
à un texte ?
# 12.17 # Comment obtenir des indices ou exposants à gauche ?
# 12.18 # Comment tracer des diagrammes commutatifs ?
# 12.19 # Comment ajuster la taille de certains opérateurs ?
# 12.20 # Comment mettre en page des algorithmes ?
# 12.21 # Comment mettre en page des formules longues ?
# 12.22 # Comment créer des unités de mesure ?
# 12.23 # Comment écrire proprement a/b ?
# 12.24 # Comment créer des notes de bas de page dans une
formule mathématique ?
# 12.25 # Comment forcer le style d'un tableau à displaystyle ?
# 12.26 # Comment réduire les espaces gérés par eqnarray ?
# 12.27 # Comment réduire la taille des indices ?
# 12.28 # Comment aligner des indices ?
# 12.29 # Comment mettre en page des théorèmes ?
# 12.30 # Comment changer la taille de la fonte ?
# 12.31 # A quoi sert la commande \displaystyle ?
# 12.32 # Comment aligner des données sur le point décimal ?
# 12.33 # Comment gérer les espaces en mode mathématique ?
# 12.34 # Comment obtenir des accolades horizontales ?
# 12.35 # Comment obtenir des points de suspensions ?
# 12.36 # Comment mettre en page un système d'équations ?
# 12.37 # Comment mettre en page des diagrammes de Feynman ?
# 12.38 # Comment obtenir une fraction ?
# 12.39 # Comment obtenir une racine ?
# 12.40 # Quels sont les délimiteurs disponibles ?
# 12.41 # Comment appeler les lettres grecques ?
# 12.42 # Quels sont les accents mathématique standards ?
# 12.43 # Comment superposer deux symboles ?
# 12.44 # Comment définir un nouvel opérateur ou symbole ?
# 12.45 # Comment obtenir des lettres grecques en gras ?

[13] RÉFÉRENCES CROISÉES
------------------------
# 13.1 # Quelles sont les commandes de base ?
# 13.2 # Comment obtenir un renvoi à une page ?
# 13.3 # Comment obtenir des références croisées à partir de
plusieurs sources ?
# 13.4 # Comment définir des liens hypertexte sous LaTeX ?
# 13.5 # Comment faire référence à ses propres compteurs ?

[14] TABLE DES MATIÈRES
-----------------------
# 14.1 # Comment générer une table des matières ?
# 14.2 # Comment générer plusieurs table des matières ?
# 14.3 # Comment ajouter une entrée dans la table des matières ?
# 14.4 # Comment changer le titre de la table des matières ?
# 14.5 # Comment changer la profondeur de la table des matières ?
# 14.6 # Comment gérer des chapitres de préface, d'introduction,
et de conclusion non numérotés ?
# 14.7 # Comment enlever la numérotation des pages de tdm ?

[15] BIBLIOGRAPHIE SOUS LATEX
-----------------------------
# 15.1 # Comment construire une bibliographie ?
# 15.2 # Comment gérer plusieurs bibliographies ?
# 15.3 # Comment changer de langue dans une bibliographie ?
# 15.4 # Comment renvoyer une référence en note de bas de page ?
# 15.5 # Comment faire référence à un document ?
# 15.6 # Comment grouper des références multiples ?
# 15.7 # Comment changer le titre de la bibliographie ?
# 15.8 # Comment changer le style de la bibliographie ?
# 15.9 # Comment construire une bibliographie à partir de
plusieurs fichiers .bib ?
# 15.10 # Comment utiliser la commande \cite dans un \caption ?
# 15.11 # Comment référencer une thèse française ou un mémoire ?
# 15.12 # Comment supprimer la virgule supplémentaire dans une
liste d'auteurs ?
# 15.13 # Comment configurer la commande \cite ?
# 15.14 # Comment construire une liste d'auteurs ?
# 15.15 # Comment spécifier un tri dans une bibliographie ?
# 15.16 # Comment référencer les pages contenant des citations ?
# 15.17 # Où trouver des styles de bibliographie ?
# 15.18 # Comment faire des références croisées ?
# 15.19 # Comment citer une URL ?
# 15.20 # Comment définir des initiales à deux lettres ?
# 15.21 # Comment conserver les majuscules dans les titres ?
# 15.22 # Comment changer l'espace entre les item ?
# 15.23 # Comment réaliser des fiches de lecture ?
# 15.24 # Comment utiliser la commande \cite dans un item ?
# 15.25 # Comment générer l'expression et al automatiquement ?

[16] INDEX
----------
# 16.1 # Quelles sont les commandes de base ?
# 16.2 # Comment construire un index hiérarchique ?
# 16.3 # Quels sont les générateurs d'index ?
# 16.4 # Comment changer le style de certains mots indexés ?
# 16.5 # Comment changer le style des pages de référence ?
# 16.6 # Comment rappeler certains mots dans un haut de page ?
# 16.7 # Comment générer plusieurs index ?
# 16.8 # Qu'est ce que IdXTeX ?
# 16.9 # Qu'est ce que xindy ?

[17] GLOSSAIRE
--------------
# 17.1 # Quelles sont les commandes de base ?
# 17.2 # Quels sont les générateurs de glossaire ?

[18] STYLES PRÉDÉFINIS
----------------------
# 18.1 # Où trouver un style de thèse ?
# 18.2 # Comment faire son CV en LaTeX ?
# 18.3 # Où trouver un format de publication ?
# 18.4 # Où trouver un style de manuel de référence ?
# 18.5 # Où trouver un style de poster ?
# 18.6 # Comment créer son propre style ?

[19] CRÉATION DE TRANSPARENTS
-----------------------------
# 19.1 # Quels sont les packages et styles existants ?
# 19.2 # Comment définir un contour oval pour des transparents ?
# 19.3 # Comment inclure des commentaires dans les transparents ?
# 19.4 # Comment modifier l'interligne sous seminar ?
# 19.5 # Comment définir des en-têtes et pieds de pages ?
# 19.6 # Comment modifier la taille du cadre d'un transparent ?
# 19.7 # Comment empêcher les figures de flotter ?
# 19.8 # Comment gérer la couleur avec seminar ?
# 19.9 # Comment imprimer des transparents en miroir ?
# 19.10 # Comment inclure une image de fond ?
# 19.11 # Comment imprimer plusieurs transparents par page ?

[20] LETTRES, MAILING ET FAXS
-----------------------------
# 20.1 # Comment structurer une lettre ?
# 20.2 # Comment préparer un mailing ?
# 20.3 # Comment faire des références dans une lettre ?
# 20.4 # Comment mettre en page un fax ?
# 20.5 # Comment positionner une adresse pour une enveloppe à
fenêtre ?
# 20.6 # Comment suppprimer la date sur une lettre ?
# 20.7 # Comment inclure une figure dans une lettre ?

[21] SYMBOLES ET LOGOS
----------------------
# 21.1 # Où trouver des symboles électroniques ?
# 21.2 # Comment dessiner des circuits électroniques ?
# 21.3 # Quelles sont les polices de symboles sous LaTeX ?
# 21.4 # Comment obtenir les symboles mâle et femelle ?
# 21.5 # Comment obtenir le symbole degré ?
# 21.6 # Où trouver des symboles astronomiques ?
# 21.7 # Où trouver une police de symboles phonétiques ?
# 21.8 # Où trouver des opérateurs de logique floue ?
# 21.9 # Comment obtenir le symbole de paragraphe ?
# 21.10 # Comment obtenir le caractère 'registered' ?
# 21.11 # Où trouver le symbole trade-mark ?
# 21.12 # Comment obtenir un underscore ?
# 21.13 # Où trouver le symbole radioactif ?
# 21.14 # Comment obtenir le logo LaTeX ?
# 21.15 # Comment obtenir le logo LateX2e ?
# 21.16 # Où trouver des chiffres entourés ?
# 21.17 # Comment obtenir le symbole numéro ?
# 21.18 # Comment obtenir les symboles pourcent et pourmille ?
# 21.19 # Comment obtenir un e dans l'o ?
# 21.20 # Quels sont les symboles réservés dans LaTeX ?
# 21.21 # Comment obtenir des lettres cursives ?
# 21.22 # Comment obtenir le logo AMS-(La)TeX ?
# 21.23 # Quels sont les symboles par défaut de LaTeX ?
# 21.24 # Quels sont les différents tirets ?
# 21.25 # Comment obtenir le symbole arobasse ?
# 21.26 # Comment obtenir un backslash ?
# 21.27 # Comment faire un carré plain de fin de démonstration ?

[22] MUSIQUE
------------
# 22.1 # Comment écrire de la musique sous LaTeX ?
# 22.2 # Comment convertir du midifile en MusicTeX ?
# 22.3 # Existe-t'il une liste de discussion de musique ?
# 22.4 # Comment éditer un livre de chants ?
# 22.5 # Comment mettre en page un programme de concert ?

[23] CONVERSIONS DE FICHIERS
----------------------------
# 23.1 # Comment générer un fichier .ps à partir d'un .dvi ?
# 23.2 # Qu'est ce que le "Literate Programming" ?
# 23.3 # Comment convertir du LaTeX en word ?
# 23.4 # Comment convertir du word en LaTeX ?
# 23.5 # Comment convertir du scribe en LaTeX ?
# 23.6 # Comment convertir du WordPerfect en LaTeX ?
# 23.7 # Comment convertir du LaTeX en RTF ?
# 23.8 # Comment convertir du RTF en (La)TeX ?
# 23.9 # Comment convertir du Excel en LaTeX ?
# 23.10 # Comment convertir du HTML en LaTeX ?
# 23.11 # Comment convertir du LaTeX en HTML ?
# 23.12 # Comment convertir un fichier dvi en ascii ?
# 23.13 # Comment convertir du WEB en LaTeX ?
# 23.14 # Comment convertir du TeX en Framemaker ?
# 23.15 # Comment enlever les balises LaTeX d'un document ?
# 23.16 # Comment convertir du SGML en (La)TeX ?
# 23.17 # Comment convertir du WinWord en LaTeX ?
# 23.18 # Comment convertir un fichier 8 bits en fichier 7 bits ?
# 23.19 # Comment convertir un fichier ChiWriter en TeX ?
# 23.20 # Où trouver une FAQ de convertisseurs (La)TeX/
Traitement de texte ?
# 23.21 # Comment convertir une image en police metafont ?
# 23.22 # Comment convertir un fichier postscript en ascii ?
# 23.23 # Comment convertir un fichier pdf en ascii ?
# 23.24 # Comment convertir du LaTeX en PDF ?
# 23.25 # Comment définir son propre format de sortie ?

[24] PREVIEWERS ET VIEWERS
--------------------------
# 24.1 # Où trouver un previewer ?
# 24.2 # Où trouver un viewer ?
# 24.3 # Comment visualiser des fichiers non postscript sous
xdvi ?

[25] LOGICIELS DE DESSINS
-------------------------
# 25.1 # Où trouver un logiciel de dessin ?
# 25.2 # Comment inclure des formules Latex dans Xfig ?
# 24.3 # Comment visualiser avec xdvi des fichier MPS générés
pas Metapost ?

[26] CORRECTEURS
----------------
# 26.1 # Où trouver un correcteur orthographique ?
# 26.2 # Où trouver un vérificateur de syntaxe LaTeX ?

[27] ASSOCIATIONS
-----------------
# 27.1 # Qu'est ce que l'association GUTenberg ?
# 27.2 # Qu'est ce que l'association TUG ?
# 27.3 # Qu'est ce que l'association AsTeX ?

[28] FONTES
-----------
# 28.1 # Que signifient les sigles T1, mf, fd etc. ?
# 28.2 # Quels sont les attributs d'une fonte ?
# 28.3 # Comment utiliser une fonte ?
# 28.4 # Comment changer la forme d'une fonte ?
# 28.5 # Comment changer la taille d'une fonte ?
# 28.6 # Comment modifier la fonte des numéros de paragraphe ?
# 28.7 # Comment modifier la fonte du mode verbatim ?
# 28.8 # Comment réaliser des changements de fontes relatifs ?
# 28.9 # Comment mettre en évidence une portion de texte ?
# 28.10 # Où trouver des fontes ?
# 28.11 # Comment suivre le chargement des fontes ?
# 28.12 # Pourquoi MakeTeXPK est lancé alors que la fonte
existe ?
# 28.13 # Comment utiliser \textsc dans un titre en conservant
le gras ?

[29] DIVERS
-----------
# 29.1 # A quoi sert la commande \par ?
# 29.2 # Comment commenter une partie d'un source LaTeX ?
# 29.3 # Comment utiliser LaTeX sur des petites machines ?
# 29.4 # Comment visualiser des paramètres ?
# 29.5 # Comment visualiser des compteurs ?
# 29.6 # A quoi servent \makeatletter et \makeatother ?
# 29.7 # Comment numéroter les lignes d'un document ?
# 29.8 # A quoi sert la commande \special ?
# 29.9 # Comment réaliser des calculs avec les variables LaTeX ?
# 29.10 # Où trouver une fonte 9pt ?
# 29.11 # Comment automatiser les compilations LaTeX ?
# 29.12 # Comment obtenir des cadres gris ?
# 29.13 # Comment obtenir certaines abbréviations ?
# 29.14 # Comment gérer les espaces après une macro ?
# 29.15 # Où trouver les notices d'utilisation des packages ?
# 29.16 # Comment obtenir des listes d'objets flottants ?
# 29.17 # Comment connaître les versions des fichiers utilisés
lors d'une compilation ?
# 29.18 # Pourquoi certaines commandes sont-elles indépendantes ?
# 29.19 # Comment installer un package ?
# 29.20 # Comment générer des codes barres ?
# 29.21 # Comment gérer des abbréviations ?
# 29.22 # Comment imprimer un fichier postscript sur une
imprimante non postscript ?
# 29.23 # Comment surcharger une commande déjà existante ?
# 29.24 # Comment reporter l'exécution d'une commande à la fin
d'une page ?
# 29.25 # Comment définir des scripts interactifs ?
# 29.26 # Comment identifier une version provisoire ?
# 29.27 # Comment obtenir des caractères barrés ?
# 29.28 # A quoi sert la commande \ensuremath ?
# 29.29 # A quoi servent les commandes \(re)newcommand ?
# 29.30 # Où trouver des hirondelles ?
# 29.31 # Comment tracer des lignes ?
# 29.32 # Comment imprimer le caractère ~ ?
# 29.33 # Comment visualiser tous les caractères d'une fonte ?
# 29.34 # A quoi sert % dans les macros ?
# 29.35 # Comment inclure l'heure dans un document ?
# 29.36 # Comment compter le nombre de mots d'un fichier ?
# 29.37 # Comment rendre inactif un caractère ?
# 29.38 # Comment utiliser le mode verbatim dans une commande ?
# 29.39 # Comment redéfinir la commande \year ?
# 29.40 # Qu'est ce que la magnification ?
# 29.41 # Comment sont gérés les postscripts dans LaTeX ?
# 29.42 # Pourquoi LaTeX n'accepte t'il pas tous les formats
d'image ?
# 29.43 # A quoi sert la commande \(re)newenvironment ?
# 29.44 # Comment récupérer le nom du fichier compilé ?
# 29.45 # Comment gérer des conditions de traitement dans un
style ?
# 29.46 # A quoi servent les commandes \setlength et
\addtolength ?
# 29.47 # Quelles sont les unités de mesure de TeX ?
# 29.48 # A quoi sert la commande \mbox ?
# 29.49 # Comment obtenir des points de suspension ?
# 29.50 # Comment désactiver une ligature ?
# 29.51 # Comment gérer les versions d'un document ?
# 29.52 # Comment changer certains titres ?
# 29.53 # Comment insérer un code source dans un document ?
# 29.54 # Comment tracer une ligne horizontale ?
# 29.55 # Comment générer un espace invisible de taille donnée ?
# 29.56 # Qu'est ce qu'une correction italique ?
# 29.57 # Quels sont les accents accessibles sous LaTeX ?
# 29.58 # Comment écrire dans un fichier pendant une compilation ?
# 29.59 # Comment gérer des compteurs ?
# 29.60 # Quels sont les différents styles de compteurs ?
# 29.61 # Comment programmer un traitement itératif ?
# 29.62 # A quoi servent les commandes savebox ?
# 29.63 # Comment résoudre certaines incompatibilités de
packages ?
# 29.64 # Combien d'internautes abonnés à une liste de
discussion faut-il pour changer une ampoule électrique (GAG)?

[30] REMERCIEMENTS
------------------


================================================================
[1] PRÉSENTATION
================================================================

# 1.1 # Quelle est l'histoire de (La)TeX ?
------------------------------------------
TeX (1978) est le formateur de texte de D. E. Knuth. A
l'origine, Knuth a développé TeX (en WEB cf. paragraphe 23.3)
notamment pour réaliser de beaux documents et écrire des
formules mathématiques.

LaTeX, écrit par L. Lamport (1982), est un jeu de macros par
dessus TeX, plus facile à utiliser que ce dernier. Il
propose notamment différents styles de document auxquels
correspondent des classes de document et une grande diversité
de macros qui répondent à divers besoins des auteurs. LaTeX a
été conçu pour rédiger des articles, des rapports, des thèses ou
des livres ou pour préparer des transparents. On peut insérer
dans le texte, des dessins, des tableaux, des formules
mathématiques et des images sans avoir à se soucier (ou presque)
de leur mise en page. Les documents produits avec LaTeX et TeX
sont d'une excellente qualité typographique.

Plain TeX écrit également par D. E. Knuth, était le premier jeu
(minimal) de macros par dessus TeX. De même Eplain, de K. Berry,
est un jeu de macros intermédiaire entre TeX et LaTeX.

Suite à une large diffusion de LaTeX beaucoup d'extensions ont
été créées par différents utilisateurs. Grâce à Murphy, ces
extensions ont introduit un certain nombre d'incompatibilités
et ont porté atteinte à la portabilité de LaTeX. C'est de cette
situation qu'est né le projet de normalisation LaTeX3, sous la
direction des gurus LaTeX : L. Lamport, F. Mittelbach, C.
Rowley, R. Schopf et tant d'autres... Pour plus de détails,
consulter :
http://www.cogs.susx.ac.uk/users/alanje/latex/latex3.html

Toutefois, pour ne pas perturber les actuels utilisateurs de
LaTeX, la version provisoire normalisée s'appelle LaTeX2e (1994)
et elle est compatible (dans la mesure du possible) avec les
anciens standards. Ainsi tous les documents écrits pour
LaTeX2.09 peuvent être compilés sous LaTeX2e en mode "LaTeX2.09
compatibility mode".

Remarque : cette compatibilité sera amenée à disparaître au fur
et à mesure des évolutions vers LaTeX3.

Les membres du projet LaTeX3 travaillent actuellement sur le
futur LaTeX3. Autrement dit, LaTeX2e ne devrait plus beaucoup
évoluer.

Il existe aussi omega, une extension 16 bits de TeX qui utilise
unicode comme représentation interne et autorise ainsi la
composition de textes multi-lingues dans les langues telles que
l'arabe, le chinois ou les langues du continent indien. Pour
plus de renseignements, vous pouvez consulter : les cahiers
GUTenbeg, TUGboat, ftp://ftp.ens.fr/pub/tex/yannis/omega,
ftp://ftp.ens.fr/pub/tex/yannis/omega-babel, ...

Signalons également NTS, un projet à très long terme qui vise
d'abord à enrichir TeX mais qui à terme, n'en gardera que les
concepts.

# 1.2 # Quels sont les principes de base de TeX ?
-------------------------------------------------
Le principe de base de TeX est la boîte ! TeX met tout dans des
boîtes et l'assemblage de ces boîtes suivant des règles données
permet de mettre en page des documents.

A l'origine, TeX a été conçu aussi bien pour créer des documents
d'une page de texte, que des documents de plusieurs centaines de
pages contenant des formules mathématiques, des tableaux, des
figures, etc. TeX travaille donc comme un imprimeur sans subir
les contraintes mécaniques inhérentes aux outils de ce dernier.
Sa précision est sans limite puisque son unité de base est le
point et que les calculs qu'il suscite sont effectués par les
ordinateurs.

Un des avantages de TeX est qu'il possède une vision globale des
choses. TeX ne se place pas uniquement au niveau du caractère
que l'auteur frappe mais aussi au niveau du mot, de la ligne, du
paragraphe, de la page ou du document dans son entier pour
évaluer ses critères de beauté. La dimension esthétique du
document est ainsi prise en compte et gérée de manière à ce
qu'elle soit maximale.

Les critères de beauté utilisés dépendent le plus souvent des
règles typographiques attachées à la langue ou au langage
employé (mathématiques par exemple), mais il peut également, à
tout moment, prendre en compte les goûts de l'auteur. TeX gère
ainsi la ponctuation, les ligatures, les coupures de mots et
les justifications verticales et horizontales.

Un autre avantage de TeX est la facilité avec laquelle il donne
accès à toutes ces possibilités de composition. En mathématiques
par exemple, il offre une quantité incroyable de symboles et
connaît leurs conventions de mise en page (taille, fonte,
espacement, etc).

Un dernier avantage couvre tous les problèmes de numérotation
des paragraphes, de tables des matières, des figures, de
références croisées, de bibliographie, d'index, etc, etc. Toutes
ces aides de lecture sont gérées automatiquement par TeX.
L'auteur a peu à s'en soucier, il n'a qu'à les déclarer.

En conclusion, vous n'avez qu'à penser au contenu de votre
document, TeX se charge du reste.

# 1.3 # Quels sont les principes de base de LaTeX ?
---------------------------------------------------
LaTeX peut être considéré comme un langage de programmation
évolué dans le sens où il s'appuie sur TeX qui est un langage de
plus bas niveau. Langage de programmation signifie également que
le document que l'on veut créer doit être décrit dans un fichier
source (.tex) puis doit être compilé. Ainsi, le compilateur
LaTeX prend en entrée un fichier source écrit en LaTeX et
produit en sortie un fichier .dvi (device independent). Ce
fichier peut ensuite être converti en fichier postscript avant
d'être imprimé. Les fichiers dvi et postscript peuvent être
visualisés à l'écran à l'aide de (pre)viewers.

L'intérêt du format dvi est qu'il permet à TeX et LaTeX d'être
indépendants du matériel qui sera utilisé pour la visualisation
ou l'impression finale du document.

Le fichier source (fichier.tex) doit comprendre un certain
nombre de commandes (balises) LaTeX qui vont permettre au
compilateur LaTeX de construire un fichier "device independent"
(.dvi). La plupart des commandes LaTeX se caractérisent par le
fait qu'elles commencent par un "backslash", que leurs
arguments obligatoires apparaissent entre accolades ({ et }) et
que leurs arguments optionnels apparaissent entre crochets ([ et
]). La structure minimale d'un rapport est en LaTeX2e est la
suivante :

Exemple :
\documentclass{classe}
\begin{document}
Votre texte...
\end{document}
%%%% fin exemple %%%%

Remarque : Le passage de LaTeX2.09 à LaTeX2e se traduit dans
l'en-tête des documents par :

\documentclass[options]{class}
\usepackage{style}
\usepackage{package}

au lieu de :

\documentstyle[options,styles,packages]{class}

ATTENTION toutefois, certains styles LaTeX2.09 ne seront pas
reconnus par LaTeX2e.

Les commandes LaTeX décrivent ainsi la classe d'un document, sa
structure, etc. Les principales classes de document disponibles
sont : 'article', 'report', 'book', 'letter' et 'slides'. Il
existe également un certain nombre d'options qui permettent de
modifier le style par défaut d'une classe (le format a4, la
taille de la fonte 12pt, etc). Ces classes et options
permettent de disposer d'une structure de base pour un
document, mais libre à vous ensuite de définir vos propres
structures grâce aux styles offerts, aux packages disponibles
notamment sur les sites CTAN (cf. paragraphe 3.1) et/ou à vos
propres macros TeX et LaTeX.

L'intérieur d'un document de classe 'article', 'report' ou
'book' est ensuite structuré grâce aux balises disponibles de
type :
\part{}, \chapter{}, \(sub)*section{}, etc. Les lettres et
les transparents font appel à d'autres structures particulières
(cf. chapitres 19 et 20).

Les informations présentées dans ces structures peuvent être
mises sous différentes formes grâce à des environnements tels
que tabular ou itemize.

Exemple :
\documentclass[12pt]{report}
\usepackage{french}

\title{Mon premier document {\LaTeX} \\
Qu'il est beau ! !}
\author{C'est moi l'auteur.}

\begin{document}

\maketitle
\tableofcontents

\part{Une partie.}
\chapter{Un chapitre.}
Texte...
\section{Une section.}
Texte...
\section{Une autre section.}
Texte...
\subsubsection{Avec une sous-section.}
Texte...
\subsubsection{Plus une autre.}
\begin{table}[htbp]
\begin{center}
\begin{tabular}{|c||c|}
\hline
donn\'ees & donn\'ees \\
\hline
\end{tabular}
\caption{Titre table. \label{table-}}
\end{center}
\end{table}
\part{Une courte deuxi\`eme partie.}
Texte...
\appendix
\chapter{Et une annexe pour finir.}
Texte...
\begin{itemize}
\item bla bla 1
\item bla bla 2
\end{itemize}
\end{document}
%%%% fin exemple %%%%

# 1.4 # Qu'est ce que le préambule du fichier source ?
------------------------------------------------------
Les appels à des packages ainsi que les définitions de
nouvelles commandes sont placées dans le préambule du document
LaTeX (i.e. entre les balises \documentstyle (LaTeX2.09) ou
\documentclass (LaTeX2e) et la commande \begin{document}).

# 1.5 # Comment faire ses premiers pas ?
----------------------------------------
En plus de la présentation faite ci-dessus quelques notions
supplémentaires sont utiles à la compréhension de LaTeX.
Celles-ci concernent essentiellement la saisie d'un texte.

Lors que l'on désire travailler dans une langue comportant des
caractères accentués LaTeX propose des saisies un peu barbares
surtout aux yeux des débutants (\'{e} pour é par exemple) mais
qui permettent de conserver la portabilité du document ainsi
saisi sur tous systèmes (caractères codés sur 7 bits). En
revanche l'utilisation de fontes contenant des caractères
accentués (codées sur 8 bits) réduit cette portabilité (pour
plus de détails lire les questions 11.3 et 28.1).

Autre remarque importante du point de vue de la gestion des
espaces et des retours chariot inclus dans le fichier
source (.tex) d'un document. LaTeX gère tout seul les espaces :
il est inutile de taper plusieurs espaces de suite entre deux
mots, ils seront transformés en un seul dans le fichier .dvi.

En outre, UN retour chariot est considéré comme un espace, sauf
s'il est suivi d'un deuxième, il marque alors la fin d'un
paragraphe et le prochain sera indenté. On peut alors sauter
autant de lignes que l'on veut dans le texte, cela n'a aucun
effet supplémentaire. Un passage à la ligne peut être forcé par
\\ ou par \newline mais dans ce cas, la première ligne du
nouveau paragraphe ne sera pas indentée. \\* empêche un saut
de page après le saut de ligne demandé. La commande \par permet
de commencer un nouveau paragraphe en laissant un espace
vertical plus important et en indentant.

Remarque : la commande \\[lgr]{} peut prendre comme paramètre une
longueur lgr pour augmenter localement un interligne.

En LaTeX, tout ce qui suit un % n'est pas lu, jusqu'au prochain
retour chariot. (Au passage le caractère % peut alors être
obtenu par \%.)

# 1.6 # Comment sont gérées les options de package ?
----------------------------------------------------
Quand on utilise plusieurs packages :
\usepackage{package1,package2}
et que l'on veut utiliser une option du package1 qui n'existe
pas pour le package2, on peut écrire :
\usepackage[option1]{package1}
\usepackage{pckge2}
Cela évite un message du type "unknown option1 for pckge2" qui
peut apparaître quand on écrit
\usepackage[option1]{package1,package2}.

On peut également écrire :
\documentclass[option1,gnagna]{article}
\usepackage{pckge1,pckge2}
qui permet de conserver l'ordre de chargement des packages mais
pas celui dans lequel seront exécutées les options par tel
package. Les options de classe sont globales et descendent à
toutes les extensions chargées si elles sont définies pour ces
extensions (certaines options sont par nature globales comme
draft, final, french, a4paper, dvips...).

# 1.7 # Quelle est la structure d'une page LaTeX ?
--------------------------------------------------
La commande \layout du package 'layout' permet de visualiser la
structure d'une page et ses différents paramètres. Globalement
elle est composée du corps du texte, d'une entête et d'un pied
de page. Des marges sont également définies de chaque côté du
corps du texte.

Exemple :
\documentclass{report}
\usepackage{layout}
\begin{document}
\layout
\end{document}
%%%% fin exemple %%%%

A chaque classe de documents sont associées différentes valeurs
aux paramètres de mise en page. L'utilisateur peut également
redéfinir ces valeurs de paramètres. Cela lui permet de gérer sa
propre mise en page. Cette FAQ donne un certain nombre de moyens
dans ce but.

Pour les documents devant être imprimés en recto-verso,
\oddsidemargin définit la marge gauche des pages impaires
(recto), et \evensidemargin la marge gauche des pages paires
(verso). Pour les documents simple face, la commande
\oddsidemargin suffit.

Les principaux paramètres d'une page sont les suivants :
+ \textheight définit hauteur du texte.
+ \textwidth définit la largeur du texte.
+ \columnsep définit l'espace entre colonnes pour un document
multi-colonnes.
+ \columnseprule définit la largeur de la ligne qui sépare les
colonnes d'un document multi-colonnes (par défaut ce paramètre
vaut 0pt i.e. pas de ligne).
+ \columnwidth définit la largeur d'une colonne. Ce paramètre
est calculé automatiquement par LateX d'après \textwidth et
\columnsep.
+ \linewidth définit la longueur de la ligne courante. Ce
paramètre est généralement utilisé dans des environnements qui
redéfinissent les marges.
+ \evensidemargin définit un espace supplémentaire dans la marge
gauche des pages paires des documents recto-verso.
+ \oddsidemargin définit cet espace pour les pages impaires d'un
document recto-verso ou pour toutes les pages dans le cas d'un
document recto uniquement.
+ \footskip définit la distance entre la dernière ligne du texte
et la première ligne du bas de page.
+ \headheight définit la hauteur de l'entête.
+ \headsep définit la distance entre la dernière ligne d'entête
et la première ligne du corps du document.
+ \topmargin définit un espace supplémentaire au dessus de
l'entête.
+ \marginparpush définit l'espace vertical minimum entre deux
notes de marge.
+ \marginparsep définit l'espace horizontal entre entre le corps
du document et les notes de marge.
+ \marginparwidth définit la largeur des notes de marge.
+ \paperheight définit la hauteur du papier sur lequel le
document sera imprimé.
+ \paperwidth définit sa largeur.

# 1.8 # Quelles sont les commandes de compilation LaTeX ?
---------------------------------------------------------
A partir d'un fichier source fichier.tex, pour générer un
fichier fichier.dvi, il faut appliquer :
latex fichier.tex
Lorsqu'une compilation échoue sur une erreur, la ligne où est
située l'erreur est indiquée. L'emplacement de l'erreur dans la
ligne est précisé par un retour à la ligne. Une explication
succincte de l'erreur est également fournie.

+ La commande ? permet alors d'avoir un menu d'aide.
+ La commande h peut permettre d'avoir une explication plus
détaillée de l'erreur sur laquelle LaTeX s'est arrêté.
+ La commande return peut permettre de forcer la suite de la
compilation.
+ La commande s permet de visualiser les messages d'erreur
suivants.
+ La commande r permet de poursuivre la compilation sans arrêt.
+ La commande q permet de continuer la compilation sans
messages.
+ La commande i permet d'insérer quelque chose (une balise
oubliée par exemple) pour pouvoir poursuivre la compilation.
+ La commande e permet d'éditer le fichier source.
+ La commande x permet d'abandonner la compilation.
+ Un chiffre de 1 à 9 permet d'ignorer les x prochains
caractères du source.

Lorsque la compilation se termine normalement, elle produit un
fichier fichier.dvi qui peut être visualisé par un utilitaire
tel que :
(UNIX) xdvi fichier.dvi

A partir d'un fichier fichier.dvi, pour générer un fichier
postscript, il faut utiliser un utilitaire tel que :
(UNIX) dvips fichier.dvi

Le fichier fichier.ps alors généré peut être imprimé. Par
exemple : (UNIX) lpr -Pimprimante fichier.ps

# 1.9 # Quels sont les fichiers utilisés par LaTeX ?
----------------------------------------------------
Il en existe différentes sortes :
+ les fichiers de compilation sont des .tex, .ltx, .toc, .lof,
.lot, .idx, .ilg, .ind, .ist, .bbl, .bib, .blg, .bst, .aux,
.dvi, .log, .texlog, .lis, .list, .ps,
+ les styles où macros sont définis dans des fichiers .cls,
.clo, .dtx, .sty, .fmt,
+ les caractères sont décrits dans des fichiers .tfm, .mf, .fd,
.pk.

# 1.10 # A quoi correspondent les messages Overfull ?
-----------------------------------------------------
Lorsque LaTeX n'arrive pas à satisfaire tous ses critères de
beauté, il peut avoir à en violer un. Cette entorse est alors
indiquée par un message de type :
Overfull \hbox (4.02349pt too wide) in paragraph at
lines 95-98
qui précise le type de dépassement, sa valeur et sa
localisation.


================================================================
[2] DOCUMENTATION
================================================================

# 2.1 # Que puis je lire sur TeX ?
----------------------------------
* "The TeXbook", de D. E. Knuth (Addison Wesley, 1984)

* "Le petit livre de TeX", de R. Seroul, (Interéditions, 1989)

* "Introduction to TeX", de N. Schwarz (Addison Wesley, 1989)

* "TeX for the beginner", de W. Snow (Addison Wesley, 1992)

* "TeX for the impatient", de P. Abrahams, K. Berry et K.
Hargreaves (Addison Wesley, 1990)

* "The advanced TeX book", de D. Salomon (Springer Verlag,
1995)

* "La maîtrise de TeX et LaTeX", de T. Lachand-Robert (Masson,
1995). Ce document peut être très utile à ceux qui veulent
programmer/comprendre/adapter des macros/packages LaTeX.

* "A TeX Primer for Scientists", de S. Sawyer et S. Krantz (CRC
Press,1995)

* "TeX by example: A Beginner's Guide", de A. Borde (Academic
Press, 1992).

* "TeX: The Program", de D. E. Knuth (Addison Wesley, 1986)

* "TeX by Topic" de V. Eijkhout est complémentaire au TeX book.
Il est disponible sur
http://www.math.ucla.edu/~eijkhout/tbt.html.

# 2.2 # Que puis je lire sur LaTeX2.09 ?
----------------------------------------
* "LaTeX, a Document Preparation System", de L. Lamport
(Addison Wesley, 1ère édition)

* "LaTeX reference manual", de L. Lamport (Addison Wesley)

* "LaTeX, Manuel utilisateur simplifié", de C. Simian (CNRS)

* "La maîtrise de TeX et LaTeX", de T. Lachand-Robert (Masson,
1995)

# 2.3 # Que puis je lire sur LaTeX2e ?
--------------------------------------
* Le fichier usrguide.tex de la distribution LaTeX décrit les
changements entre LaTeX2.09 et LaTeX2e.

* "LaTeX, a Document Preparation System", de L. Lamport
(Addison Wesley, 1994- 2nde édition)

* "The LaTeX companion", de M. Goossens, F. Mittelbach, et A.
Samarin (Addison Wesley, 1994)

* "A Guide to LaTeX2e, document preparation for beginners and
advanced users", de H. Kopla & P.W. Daly (Addison Wesley, 1995)

Remarque : très bien pour qui cherche un guide très complet. En
particulier, il met en permanence l'accent sur ce qui
est différent/spécifique entre LaTeX2e et LaTeX2.09.

* "LaTeX guide pratique - version 2e", de C. Rolland, (Addison
Wesley, 1995)

* "Joli manuel pour LaTeX2e", de B. Bayart (GEUT) à l'ESIEE. Ce
manuel est disponible sur CTAN dans /info/JMPL.ps.gz.

* "LaTeX2e, un apercu", de M. Goossens au CERN disponible sur
le WEB a http://www.loria.fr/tex.

* "Apprends LaTeX", de M. Baudoin (manuel de l'ENSTA). Cette
documentation est disponible sur le WEB ar ftp à
ftp.agm-ita.ensta.fr dans /pub/babafou/.

* "Essential LaTeX", de J. Warbrick. Ce document très
pédagogique permet de réaliser un document LaTeX en quelques
minutes. Il est disponible sur CTAN (cf. paragraphe 3.1).

* M. Herrb a traduit en français "The not so short introduction
to LaTeX2e" de T. Oetiker. La version française est disponible
par ftp à ftp.laas.fr dans /pub/Logiciels/latex/flshort/ ou sur
http://www.laas.fr/~matthieu/cours/latex2e/. Elle comprend
quelque compléments français spécifiques.

Il y a là:
flshort2e.dvi.gz Le fichier DVI
flshort2e.ps.gz Le fichier PostScript
flshort2e.tar.gz Les sources LaTeX2e

* "Objectif LaTeX", de V. Gramet et J.P. Regourd (Masson, 1995)

* "The LaTeX Graphics Companion" de M. Goossens, S. Rahtz et F.
Mittelbach. Pour une présentation plus complète, consulter :
http://www.aw.com/cp/tlgc.html.

* TeX est intégralement documenté dans le TeX book. Quand on
connaît le texbook sur le bout des doigts, on peut lire source2e
disponible à l'adresse :
http://www-fourier.ujf-grenoble.fr/~bouche/PDFtex/source2e.pdf.

# 2.4 # Que puis je lire sur AMS-LaTeX ?
----------------------------------------
* "Math into LaTeX: An Introduction to LaTeX and AMS-LaTeX" de
G. Gratzer (Birkhauser, Boston, 1996).

* Il existe également une documentation "amsldoc.tex"
disponible sur CTAN dans /latex/packages/amslatex/math/.

# 2.5 # Que puis-je lire sur la typographie ?
---------------------------------------------
* "Le développement des caractères", de H. E. Meier, (Syntax
Press, Cham, Suisse).

* "Pour une sémiologie de la typographie", de G. Blanchard,
(édité par Remy Magermans en Belgique, vendu par "Rencontres de
Lure", BP 533 71010 Macon cedex).
Il s'agit de la partie "illustrations" de la thèse de G.
Blanchard avec quand même un peu de texte... Le texte complet
n'est disponible qu'en italien: "L'eredita Gutenberg",
(Gianfranco Altieri Editore). Ouvrage fondamental...

* "Manuel de typographie élémentaire", de Y. Perousseaux,
(1995).

* "La chose imprimée", de Dreyfus et Richaudeau, (Retz, 1985).

* Lexiques des règles typographiques en usage à l'imprimerie
nationale, ISBN 2-11-081075-0 3ème édition novembre 1990.

* Voir également les sites :
http://www.csd.uwo.ca/staff/drraymon/typesetting.htm
http://ksi.cpsc.ucalgary.ca/articles/DigitalJ/DigitalJc.html
http://www.ucc.ie/info/TeX/papers/aston.html
http://www.typearts.com/
http://www.monotype.com/
http://www.rwsa.com/menu.html

* Il existe également la lettre "The Newsletter of Digital
Typography". Elle est disponible gratuitement par e-mail à
imp...@macline.com (mettre IMPRINT comme sujet).

* "The Elements of Typographic Style" de R. Bringhurst.

# 2.6 # Où trouver une bibliographie complète ?
-----------------------------------------------
http://www.loria.fr/tex/texbib.html propose une bibliographie
tenue à jour.

# 2.7 # Que puis-je lire sur les fontes ?
-----------------------------------------
* Il existe une FAQ comp.fonts disponible sur :
http://www.ora.com/homepages/comp.fonts/FAQ/

* "Metafont", de Knuth D.E. (Addison Wesley Longman).

* Voir également la note sur les fontes PostScript :
psnfss2e.tex disponible par ftp à ftp.loria.fr dans :
/pub/ctan/macros/latex/packages/psnfss/psnfss2e.tex

================================================================
[3] SITES WEB
================================================================

# 3.1 # Où trouver des infos sur le WEB ?
-----------------------------------------
* CTAN ou Comprehensive TeX Archive Network
Le système CTAN se propose de rassembler diverses informations
concernant TeX et son environnement suivant une organisation
commune. En particulier, tous les sites dits CTAN ou leurs
miroirs essaient de se synchroniser pour offrir des
informations cohérentes. En voici quelques uns :

+ France :
par ftp à ftp.loria.fr dans ./pub/ctan/ ou
par ftp à ftp.oleane.net/pub/CTAN/ (pas d'acces web. Supporte le
"quote site exec" et le tar+gz en ligne)
sur http://www.loria.fr ou
sur http://www.ens.fr/gut

+ Allemagne :
par ftp à ftp.dante.de dans ./tex-archive/ ou
par mail à mail-...@ftp.dante.de ou
sur http://www.dante.de

+ Grande-Bretagne
par ftp à ftp.tex.ac.uk dans ./tex-archive/ ou
par mail à texs...@tex.ac.uk ou
sur http://www.tex.ac.uk/UKTUG/home.html

+ Pays-Bas :
sur http://www.ntg.nl/ntg/ntg.html

+ Espagne :
sur http://gordo.us.es/Actividades/GUTH

+ États-Unis :
par ftp à ftp.cdrom.com dans /pub/tex/ctan/ ou
sur http://www.cdrom.com/pub/tex/ctan/

Une liste complète des sites CTAN peut être trouvée sur CTAN
dans /usergrps/info/usergrps.tex.

ATTENTION (08/96) : Le serveur d'archives (La)TeX aux États-Unis,
ftp.shsu.edu, n'étant plus à jour par défaut de
maintenance, il est fortement déconseillé désormais d'y
récupérer des utilitaires (La)TeX. Il reste donc les 2
serveurs CTAN << de base >>, l'anglais et l'allemand,
ainsi que leurs fidèles copies en France dont celle du
LORIA-CNRS à Nancy : ftp.loria.fr.

La plupart des sites CTAN offrent une fonctionnalité QUOTE
SITE INDEX qui permet d'accéder plus rapidement à l'information
que l'on recherche sous forme d'expression régulière. En
France, le mieux est de consulter :
http://www.loria.fr/cgi-bin/ctan-index

Remarque : il existe une liste de diffusion des annonces CTAN :
CTAN...@SHSU.edu. Pour s'y abonner, il faut envoyer
un mail à list...@SHSU.edu.

* http://www.loria.fr/tex/ présente le (La)TeX Navigator. On
peut y trouver de nombreux pointeurs (sur des docs dont cette
FAQ, des personnes, des packages, des outils LaTeX, des sites
ftp, CTAN, ...) et de nombreuses informations concernant Tex,
LaTeX, LaTeX2e, LaTeX3, AMS-LaTex, BibTeX, SliTeX, ....

* ftp : ftp.univ-rennes1.fr et
http://www.univ-rennes1.fr/ dans ./pub/GUTenberg proposent
également de nombreuses informations dont un certain nombre de
distributions pour Mac, PC et stations. Ce site présente
également un certain nombre d'archives et de publications dont
notamment les lettres et les cahiers GUTenberg.

* Le TeX macro index de D. M. Jones disponible par ftp
anonyme à theory.lcs.mit.edu dans ./pub/tex/TeX-index
recense un certain nombre de macros et styles LaTeX
actuellement existants.

* Autres sources d'informations (résumés de commandes, guides) :
+ http://www.sd.monash.edu.au/~timm/pub/guides/lshort2e.dvi
présente un guide rapide.
+ http://molscat.giss.nasa.gov/LaTeX/ présente un sommaire des
commandes LaTeX
+ http://www.tug.org/interest.html donne des pointeurs sur TeX,
LaTeX et consorts.
+ Sur CTAN dans /info/latex2e-help-texinfo.

* Un catalogue des outils TeX et LaTeX, mis à jour régulièrement
est disponible sur :
http://www.dit.csiro.au/~gjw/texpkgs.html

Ce catalogue présente une liste des packages disponibles avec
une courte description de chacun. Il est mirroré par les sites
CTAN dans /help/Catalogue/. Une version gzip de ce catalogue
est disponible à :
http://www.cdrom.com/pub/tex/ctan/help/Catalogue/
catalogue.html.gz

Voici quelques exemples de cites :
+ US CTAN :
http://www.cdrom.com/pub/tex/ctan/help/Catalogue/catalogue.html
+ UK CTAN node at:
http://www.tex.ac.uk/tex-archive/help/Catalogue/catalogue.html
+ germany
ftp://ftp.dante.de/tex-archive/help/Catalogue/catalogue.html

* http://www-h.eng.cam.ac.uk/help/tpl/textprocessing/
LaTeX_intro.html présente une introduction à LaTeX2e et aux
mathématiques.

# 3.2 # Où trouver cette FAQ ?
------------------------------
* Cette FAQ sera mensuellement postée dans fr.comp.text.tex, dans
fr.usenet.reponses et dans news.answers. Sa parution sera juste
mentionnée dans comp.text.tex.

* Une version postscript de cette FAQ est disponible sur le
LaTeX Navigator : http://www.loria.fr/tex/divers.html et à :
http://www.limsi.fr/Individu/lebourqu/latex/FAQ-LaTeX.fr.txt.

* Des versions html sont également disponibles, un grand merci
à leurs auteurs :
+ P. Girard
http://www.crt.umontreal.ca/~lab_info/latex/faq-francaise/
faq.html
+ F. Torre
http://www.lri.fr/Francais/Recherche/ia/stuff/FAQ-LaTeX
+ R. Metrich
http://www-ensimag.imag.fr/eleves/Renaud.Metrich/Unix/LaTeX.fr.html

* Elle est disponible sur CTAN dans
/help/LaTeX-FAQ-francaise/.

* Elle est également accessible sur le WEB :
+ http://www.loria.fr/tex/divers.html (LaTeX Navigator)
+ http://www.ams.org/tex/ (American Mathematical Society's TeX
Resources pages)
+ par ftp à ftp.inria.fr dans /faq/fr.comp.text.tex/
(ce site possède entre autres un miroir avec le site
rtfm.mit.edu)
+ par ftp à ftp.univ-lyon1.fr dans
/pub/faq/by-name/fr/faq-latex-francaise
+ http://diwww.epfl.ch/~jmonzani/FAQ_LaTeX.html site de l'Ecole
Polytechnique Federale de Lausanne
+ http://www.info.ucl.ac.be/~fp/texfaqfr.html.

* Par ailleurs, elle est incluse dans certaines distributions
Linux et aussi sur le CD-ROM AsTeX.

# 3.3 # Existe t'il des listes de discussion francophones ?
-----------------------------------------------------------
* Il existe la liste g...@ens.fr (de l'association GUTenberg)
dont les archives sont stockées sur :
http://www.univ-rennes1.fr/LISTES/g...@ens.fr/arc/maillist.html
et sur lesquelles on peut faire des recherches par auteurs,
sujets, etc.

+ Pour s'inscrire, envoyer un mail:
To: list...@ens.fr
From: votre_nom_d'utili...@votre.site
Subject:

subscribe gut Votre_Nom Votre_institution

+ Pour se désabonner :
mailto:list...@ens.fr
Subject:
unsubscribe gut

Voir aussi le site Web de GUTenberg, http://www.ens.fr/gut/

* Il existe également la liste om...@ens.fr. Cette liste de
diffusion concerne Omega, une extension de TeX développée par
John Plaice et Yannis Haralambous. Pour vous abonner, envoyer un
courrier électronique à list...@ens.fr, sans sujet ni
signature, dont le corps du message contient exclusivement la
ligne : subscribe omega PRENOM NOM

* La liste de diffusion « typographie » est consacrée aux
problèmes de composition, de typographie - française ou
étrangère - et de mise en pages, sans exclusive des techniques
employées : du lettrage à la main à la composition numérique en
passant par le plomb, du support papier à la page écran, du «
bon usage » du Code typographique jusqu'aux normalisations du
codage des caractères.

Ses abonnés ont en commun l'amour de la chose imprimée, y
compris sous ses formes modernes.
Pour s'inscrire, envoyez à
list...@irisa.fr
le message suivant :
SUBSCRIBE typo

# 3.4 # Où trouver d'autres FAQs LaTeX ?
----------------------------------------
* http://www.cogs.susx.ac.uk/cgi-bin/texfaq2html
* ftp://rtfm.mit.edu/pub/usenet-by-hierarchy/comp/text/tex/
* sur CTAN :
usergrps/uktug/faq
usergrps/dante/de-tex-faq
help/comp-fonts-FAQ/
* Le cahier GUTenberg numéro 23


================================================================
[4] SOURCES TEX ET LATEX
================================================================

# 4.1 # Où trouver les sources pour Unix ?
------------------------------------------
* La distribution GUTenberg est disponible par ftp à
ftp.univ-rennes1.fr dans /pub/GUTenberg/UNIX/. Elle est
basée sur MlTeX (adapté à LaTeX2e), TeX, Metafont. Les binaires
sont disponibles pour :
+ Sun4 sous solaris 2.x et SunOS 4.1.x
+ IBM RS6000 sous AIX 3 et 4
+ Silicon graphics sous irix4, 5 et 6
+ HP7xx sous Hpux9 et 10
+ HP9000 en Hpux 10
+ DECalpha sous OSF/1
+ DECstation 3100 sous Ultrix
+ PC sous Linux
+ PC i86 en Solaris 2

* Différentes distributions source Unix de TeX sont disponibles
sur CTAN dans /systems/unix/.

* La distribution teTeX pour Unix, Linux ou Irix est disponible
sur CTAN dans /systems/unix/teTeX/(distrib/binaries/). A
l'origine, cette distribution a été développée pour Linux. Il
existe également un package 'config' qui offre des fichiers de
configuration de cette distribution disponibles sur CTAN dans
/systems/unix/teTeX/contrib/.

# 4.2 # Où trouver les sources pour VMS ?
-----------------------------------------
* TeX pour VMS est disponible sur CTAN dans
/systems/vms/ puis Alpha/ ou VAX/ suivant l'architecture
utilisée.

* La distribution OpenVMS VAX et AXP de GUTenberg est disponible
par ftp à ftp.univ-rennes1.fr dans /pub/GUTenberg/vms/. Elle
s'appuie sur TeX et LaTeX2e.

# 4.3 # Où trouver les sources pour DOS, OS/2, Windows3.x/95/NT ?
-----------------------------------------------------------------
* Une distribution TeX pour PC, incluant LaTeX, BibTeX,
previewers, et drivers est disponible par ftp anonyme à
vax.eedsp.gatech.edu dans ./pub/TeX/.

* emTeX de E. Mattes, pour PC sous MS-DOS, Windows ou OS/2, est
disponible par ftp anonyme sur CTAN dans /systems/msdos/emtex.
Cette distribution inclu LaTeX, METAFONT, BibTeX, TeXcad... Il
existe également emtexgi sur CTAN dans
systems/msdos/emtex-contrib/emtexgi/ qui est une interface
Windows pour emTeX. Il existe une liste de discussion emTeX :
majo...@physik.tu-berlin.de.

* Une version emTeX francisée de M. Lavaud
(Michel...@univ-orleans.fr), distribuée par AsTeX (cf.
question 27.3) est disponible par ftp à ftp.univ-orleans.fr dans
/astex/ ou sur CTAN dans /pub/tex/PC/AsTeX/. L'installation de
la nouvelle version 2.2 est entièrement automatisée, ainsi que
la configuration des pilotes (dviscr, dviwin, dvips), GSview,
Gnuplot et les interfaces TeXShell, MicroEmacs et MenuTeX. Il y
a une fonction d'installation / desinstallation sélective avec
une vingtaine de types d'installation différents pour installer
tout ou partie de la distribution, et plusieurs modes
d'installation (ajouter, écraser, mettre à jour, désinstaller,
module par module).

Il existe même une liste de discussion AsTeX :
as...@univ-orleans.fr. Pour s'inscrire, il faut envoyer le
message HELP à list...@univ-orleans.fr.

* 4AllTeX, très complet, nécessite 4DOS (un remplacement de
command.com) pour l'utilisation de ses .btm (.bat améliorés).
4AllTeX inclut une interface (TeXelmExtel) sous windows (Tex,
LaTeX2e, BibTeX, makeindex, ...) vous trouverez le pack complet
sur ftp.loria.fr dans /systems/.

* gTeX sous MS-DOS et windows est disponible sur CTAN dans
/systems/msdos/gtex/. Son avantage est de fonctionner en 32bits
à la fois sous Windows (3.x, 95 et NT) et sous Dos via un
extender fourni. Cette distribution comprend micro-emacs,
dviwin, etc.

* Win32 MiKTeX de C. Schenk, pour windows 95 et NT, est
une version compilée de LaTeX2e (il supporte même les longs
noms de fichiers). MiKTeX est disponible par ftp à
ftp.tex.ac.uk dans /systems/win32/miktex/ou sur CTAN dans
/systems/win32/miktex/.

* DOS-GUT (distribution francisée par GUTenberg) est disponible
par ftp à ftp.univ-rennes1.fr dans /pub/GUTenberg/PC/DOS-GUT/.
Cette distribution est basée sur gtex et web2c. Elle comprend
l'éditeur TeXshell, TeX, TeX-XeT, plain, LaTeX, babel,
ArabicTeX, dviscr, dvips, metafont et GhostScript.

Remarque : la distribution DOS-GUT n'est plus activement
développée. Elle a été remplacée pas WIN-GUT.

* WIN-GUT est une version intégrée pour windows 3.1, 95 et NT
par P. Legrand. Elle permet la composition de textes français
(soit avec babel, soit avec french), américain et arabe
(ArabicTeX). Elle comprend DVIWIN, dvips, micro-emacs

* Il existe également Y&Y TeX System pour Windows. Pour plus
d'informations, vous pouvez consulter le site
http://www.YandY.com/.

* PCTeX pour DOS, Windows3.1 et Windows95/NT, qui est moins
puissant que le précédent mais qui est très simple à mettre en
place. Pour plus de détails, consulter http://www.pctex.com.

* EMTEXGI de A. Cottrell est une nouvelle distribution
disponible à
http://www.wfu.edu/Academic-departments/Economics/ftp/
emtexgi.html.

* F. Popineau a porté la version Unix de TeX (web2c 7.0, utilisé
par teTeX) sous Win32. Pour plus de détails, consulter :
ftp://ftp.ese-metz.fr/pub/TeX/win32
ou
CTAN:systems/win32/web2c

Cette distribution est native Win32, un peu plus rapide que
MikTeX, moins simple à installer (encore que ...), ne comprend
pas de previewer pour l'instant (le portage de xdvi sous win32
est en cours) mais comprend e-TeX, PDFTeX, makeindex et
letex2html (en beta).

# 4.4 # Où trouver les sources pour MacOS ?
-------------------------------------------
* CMacTeX, shareware de de T. Kiffe, comprenant TeX, tex--xet,
bibtex, makein metafont, metapost, un previewer dvi, dvips,
ps2pdf, un driver PostScript et quelques utilitaires de gestion
de fontes est disponible sur CTAN dans /systems/mac/cmactex/.
Pour plus de renseignements, consulter :
http://www.math.tamu.edu/~tkiffe/cmactex.html.

* OzTex de A. Trevorrow, est disponible par ftp à
ftp.univ-rennes1.fr dans ./pub/gut/, sur CTAN dans
/systems/mac/oztex/ ou par ftp à midway.uchicago.edu dans
./pub/OzTeX/. Cette distribution shareware intègre un previewer
dvi et un driver postscript. Elle nécessite plus de 512K de
mémoire vive.

* Direct-TeX Pro 2.1 de W. Ricken, est une distribution
shareware qui possède un environnement intégré et multi-fenêtre
très pratique, modulable de surcroît. Elle intègre TeX--XeT
3.14159 ainsi que Metafont 2.718, et tous les utilitaires qui
permettent de passer en une passe d'un source (La)TeX au .ps
final. Il y a même un éditeur intégré, quoique petit.
Direct-TeX est disponible sur CTAN dans
/systems/mac/directtex/. Elle nécessite le système 7 et 8Mo de
mémoire vive.

* Euro-Oztex de Y. Haralambous, est la distribution
proposée par GUTenberg. Cette version francisée appelle une
contribution shareware à Trevorrow et Ricken. Elle est
disponible par ftp à ftp.univ-rennes1.fr dans
/pub/GUTenberg/mac/Euro-OzTeX/.

Remarque : Euro-Oztex est une vieille version de la distribution
de GUTenberg. Elle utilisait OzTeX 1.7. Elle a été
remplacé récemment par Mac-GUT, qui elle utilise
CMacTeX.

* Mac-GUT, basée sur CMacTeX, est la distribution shareware
proposée par GUTenberg (donc francisée). Elle n'est disponible
à l'heure actuelle que sur le cd-rom TeX-Live distribué par
GUTenberg à ses adhérents. Pour plus de détails, consulter :
http://www.tug.org/texlive.html.

* Pour d'autres renseignements, voir la "Macintosh TeX/LaTeX
Software Page" : http://www.esm.psu.edu/mac-tex/.

# 4.5 # Où trouver d'autres sources ?
-------------------------------------
* Atari
TeX pour Atari ST est disponible par ftp à
atari.archive.umich.edu dans ./atari/tex/ ou à
ifi.informatik.uni-stuttgart.de dans ./pub/atari.st/tex/ ou sur
CTAN dans /systems/atari/. Pour tous renseignements contacter
at...@atari.archive.umich.edu par un mail "help".

* Amiga
+ PasTeX, implémentation de TeX 3.1 et METAFONT 2.7 sont
disponibles par ftp anonyme à merlin.etsu.edu dans
/ab20/AMIGA/ ou à forwiss.uni-passau.de dans
/pub/amiga/tex ou sur CTAN dans /systems/amiga/.

+ On peut trouver également PasTeX 1.4 sur les mirroirs FTP
aminet (sunsite.cnam.fr. ftp.grolier.fr, ftp.netnet.net, ...)
dans le répertoire /pub/aminet/text/tex/. PasTeX 1.4 est le
portage de LaTeX2e. La distribution comprend également dvips.

+ XFig est disponible dans /pub/aminet/gfx/edit/.

+ Ghostscript est disponible dans /pub/aminet/gfx/show/.

+ Une version LaTeX2e a également été développée à l'ESIEE
http://www.esiee.fr/~tex/Install/Amiga/index.html.

* Tandy 6000
Pour tous renseignements contacter Ken Yap
(k...@syd.dit.csiro.au).

* TOPS-20
Une distribution TeX sur TOPS-20 est disponible par ftp
anonyme à ftp.math.utah.edu dans ./pub/tex/pub/web.


================================================================
[5] ÉDITER LATEX
================================================================

# 5.1 # Quel éditeur de texte puis-je utiliser ?
------------------------------------------------
* (X-)Emacs est un éditeur sous Unix qui offre en standard un
mode d'édition, un peu fruste mais néanmoins pratique,
facilitant la composition de documents (La)TeX. Une extension à
emacs, AUC-TeX (disponible sur CTAN dans /support/auctex/ ou à
http://sunsite.auc.dk/auctex/), fournit de nombreuses facilités
supplémentaires (indentation automatique, messages d'erreur en
anglais compréhensible, gestion des documents multi-fichiers,
etc.)

Emacs reconnaît automatiquement certaines extensions
(tex,sty...) dans un nom de fichier, et active le mode en
question automatiquement. Si votre fichier n'est pas reconnu
comme un document (La)TeX, vous pouvez spécifier sur la première
ligne de votre fichier : % -*-latex-*-

Les packages 'font-lock' et 'hilit19' ('hilit319' pour Xemacs)
ou plus récent 'font-latex' (basé sur font-lock), permettant de
choisir les couleurs et les polices mettant en évidence la
syntaxe d'un fichier, sont utilisables avec les modes (La)TeX.
Pour plud de détails, lire :
ftp://ftp.loria.fr/pub/ctan/support/ultratex/.

Le package (standard) 'imenu' donne accès à un menu listant les
en-têtes de section du document, et permet de retrouver
celles-ci facilement dans un grand document. Une extension à ce
mécanisme permet de mieux visualiser la structure du document,
en indentant les sous-sections.

* GNU emacs et AUCTeX peuvent également être utilisés sous
MS-DOS ou OS/2.

* JED est un clone multi-plateformes proche d'emacs. JED est en
fait un emacs allégé qui offre des facilités dans l'édition de
fichier (La)TeX. Il tourne sous Unix/VMS/Dos/Win. Il est
disponible à http://space.mit.edu/~davis.

* Xcoral, éditeur sous Unix offre des fonctionnalités de même
type que emacs mais non interactives. Il est disponible par ftp
à ftp.inria.fr dans /X/contrib-R5/clients/ ou à ftp.x.org dans
/contrib/.

Xcoral est un éditeur multi-fenêtres pour X Window System,
offrant un certain nombre de facilités notamment pour écrire
des programmes perl, ada, fortran, C, C++, java ainsi que des
documents Latex ou Html.

Cet éditeur comprend un interpréteur Ansi C 'built-in' qui
permet aux utilisateurs d'étendre ses fonctionnalités
facilement. Un manuel d'aide complet indexé est disponible
on-line.

* Alpha est un éditeur Macintosh shareware assez proche d'emacs.
Il est disponible entre autres par ftp à
ftp://www.cs.umd.edu/pub/faculty/keleher/Alpha/.

Cet éditeur est hautement configurable grâce à un langage de
programmation intégré, Tcl. Il possède entre autres un mode
LaTeX très convivial. Alpha permet aussi une interaction avec le
compilateur (OzTeX ou Direct-TeX) en lançant la compilation
d'une combinaison de touches. La dernière version d'Alpha est la
6.5. Elle comprend la version 3.2 des macros freeware Alpha
LaTeX de T. Scavo.

Pour plus de renseignements, voir :
+ http://www.cs.umd.edu/~keleher/alpha.html
+ http://www.cs.umd.edu/~keleher/latex_guide/latex_guide.html

* MicroEmacs (inclu dans la distribution DOS-GUT), sous windows,
permet d'éditer et de gérer des documents TeX.

* Eddi4TeX, sous MS-DOS ou OS/2, est un éditeur spécifiquement
conçu pour TeX, il offre la couleur, vérifie la syntaxe. Il
est disponible sur CTAN dans /systems/msdos/e4t/ ou
/systems/OS2/epmtex/.

* Scientific Word pour windows est un éditeur qui permet presque
de visualiser un document LaTeX en WYSIWYG (What You See Is What
You Get). Il facilite l'édition d'un document LaTeX en
permettant une insertion aisée des symboles, l'édition de
tableau, ... par le "mulot" ou par raccourcis clavier. Pour
l'utiliser pleinement, il vaut mieux connaître LaTeX.

Remarque : ce produit est commercial et cher.

* LyX est un traitement de texte sous X11 qui offre une sortie
LaTeX2e. Il est presque WYSIWYG. LyX présente les avantages
d'être petit, rapide et gratuit. LyX est encore en
développement. Une version NON définitive est disponible à
http://www-pu.informatik.uni-tuebingen.de/users/ettrich ou
par ftp à sunsite.unc.edu dans /pub/Linux/X11/xapps/editors ou
par ftp à ftp.via.ecp.fr dans /pub/lyx/ ou
WWW http://www.lehigh.edu/~dlj0/LyriX.html
Il existe également une liste de discussion : l...@via.ecp.fr.

Parmi les fonctionnalités, on trouve:
- éditeur d'équations,
- éditeur de tables,
- inclusion d'images au format EPS,
- correction d'orthographe,
- etc.

Les dernières docs sont disponibles par ftp à ftp.via.ecp.fr
dans /pub/lyx/doc/.

* TeXShell, écrit par J. Schlegelmilch, est disponible sur CTAN
dans /systems/msdos/texshell/ts271.zip. C'est un éditeur Windows
sous dos qui offre une coloration des mots clés LaTeX, une aide
en ligne et d'autres petites fonctionnalités telles que les
compilations associées à des boutons. La distribution DOS-GUT
utilise TeXshell et offre un MicroEmacs francisé.

Il existe également TeXShell pour X Window system (Tcl/Tk)
disponible par ftp à sunsite.unc.edu dans
/pub/Linux/apps/tex/tsYYMMDD.tgz.

* WINEDT95,logiciel Shareware pour Windows 95, est un éditeur
avec menu TeX/LaTeX qui permet de repérer les commandes LaTeX et
de compter les délimiteurs. Il est disponible sur CTAN dans
/support/winedt/. Il est pourvu d'un correcteur orthographique.

* wintex95 disponible sur CTAN dans /systems/win32/wtex95/ est
un éditeur flexible offrant un coloriage automatique de la
syntaxe LaTeX. Il offre également une complétion automatique,
et des palettes de symboles, un éditeur de tableaux, des
touches de raccourci, le lancement de programmes externes, etc.
C'est un shareware.

* Cicero est un traitement de texte sous X11R6 et Motif2.0. Le
package comprend des fontes X postscript, ghostscript, TeX
(dvips) et Cicero. Pour plus de renseignements, consulter :
http://zeus.informatik.uni-frankfurt.de/~weis/cicero.html

* X-Window Shell pour TeX (OpenLook ou Xaw/Xaw3d) disponible
sur
http://www.tm.bi.ruhr-uni-bochum.de/personal/marc/xtexshell.html.

* xtem: une interface graphique offrant des fonctionnalités
TeX/LaTeX disponible sur
http://ftp.lrw.uni-bremen.de/xtem/xtem_texmenu.html.

* STEAD "Sympathetic Tk-based Editor for Average Dummies" sous
Unix (ressemble à Alpha sur mac) est disponible par ftp à
ftp.ensta.fr dans /pub/tcl/contrib_ensta/. La particularité de
cet éditeur est qu'il est lui même écrit en texte pur ! En
effet, il est entièrement écrit en langage TCL et TK (son
extension permettant d'écrire par script une application avec
bouton, menu, ... sous Xwindows). Une conséquence est que cet
éditeur nécessite la présence de l'interpréteur TCL (appelé wish
et disponible sur ftp.ibp.fr fichier
/pub/tcl/distrib/tk3.6.tar.gz (il y a tout ce qu'il faut avec :
la doc, des exemples, ...) Le répertoire contient également le
projet du livre sur TCL...

STEAD est un éditeur simple d'utilisation, contextuel et
configurable.

Autre avantage : une fois l'interpréteur disponible, l'éditeur
tourne aussi bien sur SUN3 (pas vite !), SUN4, HP, et même sur
les PC avec UNIX BSD.

* Funtek, de V. Vidal, sous X-Windows system et Motif, est un
éditeur texte orienté latex, il traite le source latex page à
page, permet un accès aux symboles spéciaux, et une construction
de tableaux automatique. La version actuelle est une bêta.

* D'autres éditeurs sous dos, Mac et windows95 sont présentés
dans http://www.jumbo.com.

# 5.2 # Qu'est ce que Lollipop ?
--------------------------------
C'est un jeu de macro de V. Eijkhout, destiné à faciliter
l'écriture de macros TeX. Il est disponible sur CTAN dans
/macros/lollipop/.


================================================================
[6] GESTION DE LA MISE EN PAGE
================================================================

# 6.1 # Comment modifier l'interligne d'un document ?
-----------------------------------------------------
* Pour modifier l'espace interligne d'un document on peut
utiliser la commande \linespread (solution non recommandée).
Par exemple, \linespread{1.6} permet de doubler l'intervalle
par défaut.

* \renewcommand{\baselinestretch}{1.2} placé dans le
préambule permet d'obtenir le même résultat pour tout le
document.

* Il existe également les packages 'doublespace' (pour
LaTeX2.09) et 'setspace' (pour LaTeX2e) disponibles sur CTAN
dans /macros/latex2.09/contrib/misc/ pour l'un et
/macros/latex/contrib/other/misc/ ou
/macros/latex/contrib/supported/setspace/ pour l'autre.
setspace.sty définit les environnements singlespace,
onehalfspace et doublespace. L'utilisation de ces styles est
recommandée parce que plus robuste (gestion des tableaux, des
notes de bas de page, ...).

* Pour réduire l'interligne d'un paragraphe (à celui de small
par exemple) sans modifier la taille de la fonte on peut
utiliser \small{\normalsize texte à interligne réduit}\par ou
encore {\advance\baselineskip -1pt le texte \par}.

* Localement, on peut également utiliser la commande
\baselineskip de la manière suivante :

Exemple :
{\setlength{\baselineskip}{1.2\baselineskip}
Texte affecté
\par} %%% <= terminer le paragraphe
%%%% fin exemple %%%%

# 6.2 # Comment gérer un document recto-verso ?
-----------------------------------------------
* En LaTeX 2.09 il faut passer twoside comme option de la
commande documentstyle. Ensuite, pour forcer les entêtes de
chapitre à commencer sur une page impaire, il faut inclure la
commande \cleardoublepage avant chaque début de chapitre.

Exemple :
\documentstyle[twoside]{report}

\begin{document}
\cleardoublepage
\chapter{Introduction.}
Texte.

\cleardoublepage
\chapter{Thèse.}
Texte.

\end{document}
%%%% fin exemple %%%%

* Il existe également la macro suivante qui redéfinit la
commande \cleardoublepage pour que les pages insérées soient
vides (i.e. sans entête ni bas de page).
%%%% debut macro %%%%
% whitecdp (formerly schulzrinne.sty) --provide for blank pages
% between chapters
% This redefinition of the \cleardoublepage command provides
% for a special pagestyle for the "extra" pages which are generated
% to ensure that the chapter opener is on a recto page.
% The pagestyle is "chapterverso"; for many publishers, this should be
% identical to "empty", so that's the default.
\def\cleardoublepage{\clearpage
\if@twoside
\ifodd\c@page\else
\null\thispagestyle{chapterverso}\newpage
\if@twocolumn\null\newpage\fi
\fi
\fi
}%
\def\ps@chapterverso{\ps@empty}%
%%%% fin macro %%%%

* LaTeX2e prévoit directement les options de classe twoside et
openright.

Exemple :
\documentclass[twoside,openright]{report}
%%%% fin exemple %%%%

# 6.3 # Comment modifier le style des titres ?
----------------------------------------------
* Les définitions de \section, \sub(sub)section, etc, se
trouvent dans les fichiers .cls (report.cls, article.cls,
book.cls).

ATTENTION : Il est vivement conseillé de ne pas modifier
directement ces classes mais de redéfinir
un fichier.sty avec les nouvelles commandes ou
d'utiliser \makeatletter et \makeatother.

Exemple :
La syntaxe de définition d'une nouvelle section est :
\renewcommand\section{\@startsection {section}{1}{\z@}%
{-3.5ex \@plus -1ex \@minus -.2ex}%
{2.3ex \@plus.2ex}%
{\reset@font\Large\bfseries}}

Explication :
+ La commande \@startsection permet de gérer : la table des
matières, la numérotation des titres, les références,
les titres des sections dans l'en-tête, etc...
+ {section} indique qu'il s'agit d'une section
+ {1} indique son niveau dans la table des matières
+ {\z@} indique son niveau d'indentation (zéro)
+ {-3.5ex \@plus -1ex \@minus -.2ex} définit l'espace qui sera
ajouté au dessus du titre
+ {2.3ex \@plus.2ex} définit l'espace qui sera ajouté en dessous
du titre. Si ce nombre est négatif alors il s'agit d'un
espacement horizontal, pour avoir des titres "en ligne",
comme ça :
Titre de ma section. Nous allons parler... bla,
bla, bla...
+ \@plus et \@minus permettent de jouer sur l'élasticité de ces
espaces
+ {\reset@font\Large\bfseries} sont les commandes de mises en
forme du titre.
%%%% fin exemple %%%%

* Pour augmenter l'espace avant une section il suffit, par
exemple, d'écrire, dans le préambule du document :

%%%% debut macro %%%%
\makeatletter
\renewcommand\section{\@startsection{section}{1}{\z@}%
{2cm \@plus -1ex \@minus -.2ex}%
{2.3ex \@plus.2ex}%
{\reset@font\Large\bfseries}}
\makeatother
%%%% fin macro %%%%

* Le package 'sfheaders' de M. Loreti, ci dessous met les titres
dans une fonte sans serif quelle que soit la classe de
document utilisée. Pour l'utiliser, il suffit de sauvegarder les
macros suivantes dans un fichier SFheaders.sty et d'appeler
\usepackage{SFheaders}.

%%%% debut macro %%%%
% Package: SFheaders.sty
% Sans-Serif headers; modified from {book|report|article}.cls
% defaults.
% Last modified: MLO 1997-05-06
%
% Author: Maurizio Loreti, aka MLO or (HAM) I3NOO
% Work: University of Padova - Department of Physics
% Via F. Marzolo, 8 - 35131 PADOVA - Italy
% EMail: lor...@padova.infn.it
% WWW: http://wwwcdf.pd.infn.it/~loreti/mlo.html

\NeedsTeXFormat{LaTeX2e}
\ProvidesPackage{sfheaders}[1997/05/06 Sans-Serif headers]

\@ifclassloaded{article}{
% Here if \documentclass{article}
\def\@part[#1]#2{%
\ifnum \c@secnumdepth >\m@ne
\refstepcounter{part}%
\addcontentsline{toc}{part}{\thepart\hspace{1em}#1}%
\else
\addcontentsline{toc}{part}{#1}%
\fi
{\parindent \z@ \raggedright
\interlinepenalty \@M
\normalfont
\ifnum \c@secnumdepth >\m@ne
\Large \sffamily \bfseries \partname~\thepart
\par\nobreak
\fi
\huge \sffamily \bfseries #2%
\markboth{}{}\par}%
\nobreak
\vskip 3ex
\@afterheading}
\def\@spart#1{%
{\parindent \z@ \raggedright
\interlinepenalty \@M
\normalfont
\huge \sffamily \bfseries #1\par}%
\nobreak
\vskip 3ex
\@afterheading}}
{\@ifclassloaded{book}{
% Here if \documentclass{book}
\def\@part[#1]#2{%
\ifnum \c@secnumdepth >-2\relax
\refstepcounter{part}%
\addcontentsline{toc}{part}{\thepart\hspace{1em}#1}%
\else
\addcontentsline{toc}{part}{#1}%
\fi
\markboth{}{}%
{\centering
\interlinepenalty \@M
\normalfont
\ifnum \c@secnumdepth >-2\relax
\huge \sffamily \bfseries \partname~\thepart
\par
\vskip 20\p@
\fi
\Huge \sffamily \bfseries #2\par}%
\@endpart}

\def\@spart#1{%
{\centering
\interlinepenalty \@M
\normalfont
\Huge \sffamily \bfseries #1\par}%
\@endpart}

\def\@makechapterhead#1{%
\vspace*{50\p@}%
{\parindent \z@ \raggedright \normalfont
\ifnum \c@secnumdepth >\m@ne
\if@mainmatter
\huge \sffamily \bfseries \@chapapp\space \thechapter
\par\nobreak
\vskip 20\p@
\fi
\fi
\interlinepenalty\@M
\Huge \sffamily\ bfseries #1\par\nobreak
\vskip 40\p@
}}

\def\@makeschapterhead#1{%
\vspace*{50\p@}%
{\parindent \z@ \raggedright
\normalfont
\interlinepenalty\@M
\Huge \sffamily \bfseries #1\par\nobreak
\vskip 40\p@
}}
}{
% Here if none of the above (\documentclass{report} ?)
\def\@part[#1]#2{%
\ifnum \c@secnumdepth >-2\relax
\refstepcounter{part}%
\addcontentsline{toc}{part}{\thepart\hspace{1em}#1}%
\else
\addcontentsline{toc}{part}{#1}%
\fi
\markboth{}{}%
{\centering
\interlinepenalty \@M
\normalfont
\ifnum \c@secnumdepth >-2\relax
\huge \sffamily \bfseries \partname~\thepart
\par
\vskip 20\p@
\fi
\Huge \sffamily \bfseries #2\par}%
\@endpart}

\def\@spart#1{%
{\centering
\interlinepenalty \@M
\normalfont
\Huge \sffamily \bfseries #1\par}%
\@endpart}

\def\@makechapterhead#1{%
\vspace*{50\p@}%
{\parindent \z@ \raggedright \normalfont
\ifnum \c@secnumdepth >\m@ne
\huge \sffamily \bfseries \@chapapp\space \thechapter
\par\nobreak
\vskip 20\p@
\fi
\interlinepenalty\@M
\Huge \sffamily \bfseries #1\par\nobreak
\vskip 40\p@
}}

\def\@makeschapterhead#1{%
\vspace*{50\p@}%
{\parindent \z@ \raggedright
\normalfont
\interlinepenalty\@M
\Huge \sffamily \bfseries #1\par\nobreak
\vskip 40\p@
}}
}}

\renewcommand{\section}{\@startsection {section}{1}{\z@}%
{-3.5ex \@plus -1ex \@minus -.2ex}%
{2.3ex \@plus.2ex}%
{\normalfont\Large\sffamily\bfseries}}

\renewcommand{\subsection}{\@startsection{subsection}{2}{\z@}%
{-3.25ex\@plus -1ex \@minus -.2ex}%
{1.5ex \@plus .2ex}%
{\normalfont\large\sffamily\bfseries}}

\renewcommand{\subsubsection}{\@startsection{subsubsection}{3}%
{\z@}%
{-3.25ex\@plus -1ex \@minus -.2ex}%
{1.5ex \@plus .2ex}%
{\normalfont\normalsize\sffamily\bfseries}}

\renewcommand{\paragraph}{\@startsection{paragraph}{4}{\z@}%
{3.25ex \@plus1ex \@minus.2ex}%
{-1em}%
{normalfont\normalsize\sffamily\bfseries}}

\renewcommand{\subparagraph}{\@startsection{subparagraph}{5}%
{\parindent}%
{3.25ex \@plus1ex \@minus .2ex}%
{-1em}%
{\normalfont\normalsize\sffamily\bfseries}}
\endinput

%%
%% End of `SFheaders.sty'.
%%%% fin macro %%%%

* Le package 'fncychap' disponible sur CTAN dans
/macros/latex/contrib/supported/fncychap/ propose un ensemble
d'entêtes de chapitre prédéfinies.

# 6.4 # Comment obtenir un document multicolonnes ?
---------------------------------------------------
* L'option standard twocolumn permet de présenter un texte sur
deux colonnes verticales.

Exemple :
\documentclass[twocolumn]{article}
\usepackage{french}

\begin{document}

Voici un texte sur deux colonnes que \LaTeX{} n'équilibre pas
par lui-même (il remplit les colonnes les unes après les
autres). L'espace entre les colonnes peut être modifié comme
indiqué plus loin. Une ligne de séparation des colonnes peut
également être insérée.

\end{document}
%%%% fin exemple %%%%

* Pour agir localement, on peut utiliser les commandes :
\twocolumn[texte sur une colonne]{texte sur deux colonnes}
puis \onecolumn{Texte sur une colonne} ou plus généralement
\twocolumn et \onecolumn.

Pour une meilleure lisibilité du source, on peut également
utiliser les environnements correspondants.

Exemple :
\documentclass{article}
\usepackage{french}

\begin{document}

Dans la classe article, le texte est, par défaut, mis en page
sur une seule colonne. Il est toutefois possible de passer
temporairement sur deux colonnes.

\twocolumn[Un titre sur une colonne, un peu long pour le
prouver.]{Et voici enfin un texte sur deux colonnes~; comme
promis~! Encore une fois, \LaTeX{} n'équilibre pas le
remplissage des deux colonnes si bien qu'il est obligé pour
chaque changement de colonnage de changer de page.}

\onecolumn
Ceci permet de repasser sur une colonne pour la suite du
document.

\begin{twocolumn}
Encore quelques mots sur deux colonnes. Même si le texte est
trop court pour voir apparaître la deuxième colonne.
\end{twocolumn}
\end{document}
%%%% fin exemple %%%%

* Le package 'multicol', disponible sur CTAN dans
/macros/latex/packages/tools/, définit l'environnement multicols
qui permet de redéfinir localement le nombre de colonnes
désirées (10 maximum). Lorsqu'une page n'est pas complète, le
texte apparaît réparti sur toutes les colonnes. Ainsi chaque
changement de colonnage n'entraîne plus un changement de page.

Exemple :
\documentclass{article}
\usepackage{multicol}
\usepackage{french}
\setlength{\columnseprule}{0.5pt}
\begin{document}

\begin{multicols}{3}[Titre sur une seule colonne.]
3~colonnes équilibrées, 3~colonnes équilibrées, 3~colonnes
équilibrées, 3~colonnes équilibrées
\end{multicols}

\begin{multicols}{2}[\section{Titre numéroté.}]
blabla sur deux colonnes, c'est plus sérieux. C'est le
style qui est généralement utilisé pour écrire des
articles.
\end{multicols}
\end{document}
%%%% fin exemple %%%%

Pour ajouter un titre numéroté qui apparaisse sur toute la
largeur de la page, il faut utiliser l'option [\section{Titre.}]
juste après \begin{multicols}{nb-col}.

Remarques :
+ Pour qu'une ligne de séparation apparaisse entre les
colonnes, il faut utiliser : \setlength{\columnseprule}{1pt}.

+ Pour redéfinir la largeur de l'espace inter-colonnes, il faut
utiliser \setlength{\columnsep}{30pt}.

# 6.5 # Comment composer une brochure ?
---------------------------------------
* Pour redéfinir un format de page, (par exemple un A4 plié en
trois), il faut utiliser la commande \setlength. Il suffit de
savoir quelles sont les longueurs à préciser, le meilleur moyen
pour ce faire est de les visualiser avec la commande \layout
(définie par le package 'layout' disponible sur CTAN dans
/macros/latex/packages/tools/).

%%%% debut macro %%%%
% (Th. Bouche)
\ProvidesPackage{a6size}
% rien a voir avec la taille : ajustement du \baselineskip
\renewcommand\normalsize{%
\@setfontsize\normalsize\@xiipt{13.5}%
\abovedisplayskip 12\p@ \@plus3\p@ \@minus7\p@
\abovedisplayshortskip \z@ \@plus3\p@
\belowdisplayshortskip 6.5\p@ \@plus3.5\p@ \@minus3\p@
\belowdisplayskip \abovedisplayskip
\let\@listi\@listI}
\renewcommand\small{%
\@setfontsize\small\@xipt{12.4}%
\abovedisplayskip 11\p@ \@plus3\p@ \@minus6\p@
\abovedisplayshortskip \z@ \@plus3\p@
\belowdisplayshortskip 6.5\p@ \@plus3.5\p@ \@minus3\p@
\def\@listi{\leftmargin\leftmargini
\topsep 9\p@ \@plus3\p@ \@minus5\p@
\parsep 4.5\p@ \@plus2\p@ \@minus\p@
\itemsep \parsep}%
\belowdisplayskip \abovedisplayskip}
\normalsize

\setlength\paperheight {148mm}%
\setlength\paperwidth {105mm}%
%\voffset-1cm
%\hoffset-2cm
\setlength{\topmargin}{-1.3cm}%
\setlength{\oddsidemargin}{-.5cm}%
\setlength{\evensidemargin}{-1cm}%
\setlength{\marginparsep}{0\p@}%
\setlength{\headsep}{0\p@}%
% calcule la hauteur du texte en fonction du \baselineskip, pour
% que les lignes soit placees au meme niveau sur toutes les pages
\setlength{\textheight}{\topskip}
\addtolength{\textheight}{22\baselineskip}%
\setlength{\textwidth}{7cm}%
\setlength{\footskip}{23\p@}% (originellement : 48)
%\setlength{\baselineskip}{13\p@}%
%\setlength{\marginparwidth}{0\p@} %
%\addtolength{\baselineskip}{.2\baselineskip}%
\setlength{\parindent}{0\p@}
%\addtolength{\headsep}{\headsep}
%\setlength{\push@skip}{.2\textwidth}
\newenvironment{page}{\vspace*{\stretch{1}}}
{\vspace*{\stretch{2.5}}\newpage}
\pagestyle{plain}
%%%% fin macro %%%%

Il faut ensuite opérer ce que les imprimeurs appellent une
imposition : imprimer la page tant à tel endroit de la x-ième
feuille de telle sorte qu'il n'y ait plus qu'à plier la liasse
pour obtenir un livre prêt à être relié. Cette étape est facile
à réaliser soit à l'aide de dvidvi (mais qui n'autorise pas les
rotations, ce qui peut en limiter l'intérêt pour des formats
spéciaux) soit avec pstops :
pstops "2:0L@.7(21cm,0)+1L@.7(21cm,14.85cm)" un.ps deux.ps
disponible sur CTAN dans /support/psutils/.

* L'option a5paper de LaTeX2e permet également de faire cela.
Il faut ensuite utiliser dvidvi et dvips en -t landscape. dvips
est disponible entre autres par ftp à ftp.univ-rennes1.fr dans
/pub/GUTenberg/sources/.

* Le package 'a5booklet' est disponible sur CTAN dans
/pub/tex/dviware/a5booklet/.

* Consulter également le package '2up' pour LaTeX2e.

* Le package 'poligraf' disponible sur CTAN dans
/macros/TeX-PS/cmyk-hax/ permet d'agir sur la mise en page d'un
document avant impression.

* psnup et psbook peuvent également permettre de faire de la
composition mais il vaut alors mieux travailler avec des polices
postscript. Ces utilitaires sont accessibles via :
http://www.math.tamu.edu/~tkiffe/cmactex.html.

Exemple (J.A. Ferrez) :
=================SNIP
#!/bin/sh
#
# Turn a (clean) PS file into a booklet
#
# input on stdin or file in arg
#
# psbook -- reorder the pages
# see -s option for _very_ large files
# psnup -- scale and place two pages onto one
# pstops -- reverse the odd pages for duplex printing
#
# outup on stdout
#
psbook $1 | psnup -2 | pstops "2:0,1U(21cm,29.7cm)"
=================SNIP
%%%% fin exemple %%%%

* Le résultat est possible également en ne travaillant que sur
le postscript : dvips -h twoup -t landscape fichier.dvi.

* Sur PC on peut utiliser dvidrv.

* Pour PC twoup fait cela, mais ce n'est pas du domaine public.

# 6.6 # Comment définir un format de document ?
-----------------------------------------------
La structure d'une page LaTeX permet de nombreux ajustements :
+ \setlength{\textwidth}{??cm} permet de fixer la largeur du
texte
+ \setlength{\textheight}{??cm} permet de fixer la hauteur du
texte
+ \setlength{\oddsidemargin}{(-)??cm} permet de définir la
marge gauche des pages impaires
+ \setlength{\evensidemargin}{(-)??cm} permet de définir la
marge gauche des pages paires
+ \setlength{\topskip}{??cm} laisse un espace en haut de page
+ \setlength{\footskip}{??cm} laisse un espace en bas de page
+ \setlength{\headheight}{??cm} fixe la hauteur de l'entête
+ \addtolength{\topmargin}{(-)??cm}
+ \addtolength{\textheight}{(-)??cm}

* Il existe également les packages 'a4' et 'a4wide' disponibles
sur CTAN respectivement dans
/macros/latex/contrib/supported/ntgclass/ et dans
/macros/latex/contrib/other/misc/ qui permettent de redéfinir
les marges et largeur de texte.

* Le package 'geometry' est plus flexible que les précédents
pour définir son propre format de page. Il définit de
nouvelles variables de structure de la page. Il propose en
outre un ensemble de formats par défaut (a0paper, b5paper,
legalpaper, ...).

# 6.7 # Comment redéfinir les marges d'un document ?
----------------------------------------------------
* Le package 'vmargin', de V. Kuhlmann, est disponible sur CTAN
dans /macros/latex/contrib/other/misc/. Il permet facilement de
redéfinir globalement (pour tout le document) les marges d'un
document par la commande :
\setmarginsrb{1}{2}{3}{4}{5}{6}{7}{8}
1 est la marge gauche
2 est la marge en haut
3 est la marge droite
4 est la marge en bas
5 fixe la hauteur de l'entête
6 fixe la distance entre l'entête et le texte
7 fixe la hauteur du pied de page
8 fixe la distance entre le texte et le pied de page

* L'environnement changemargin décrit ci-dessous permet de
modifier localement les marges d'un document. Il prend deux
arguments, la marge gauche et la marge droite (ces arguments
peuvent prendre des valeurs négatives).

%%%% debut macro %%%%
\newenvironment{changemargin}[2]{\begin{list}{}{%
\setlength{\topsep}{0pt}%
\setlength{\leftmargin}{0pt}%
\setlength{\rightmargin}{0pt}%
\setlength{\listparindent}{\parindent}%
\setlength{\itemindent}{\parindent}%
\setlength{\parsep}{0pt plus 1pt}%
\addtolength{\leftmargin}{#1}%
\addtolength{\rightmargin}{#2}%
}\item }{\end{list}}
%%%% fin macro %%%%

Exemple :
\begin{changemargin}{2cm}{-1cm}
Ceci permet d'augmenter la marge gauche de 2cm et de diminuer
celle de droite de 1cm.
\end{changemargin}
A comparer avec un texte qui occupe toute la largeur de la page,
comme celui-ci.
%%%% fin exemple %%%%

* Le package 'geometry' permet de redéfinir les marges d'un
document ou de définir le layout de la page.

Exemple 1 :
\geometry{margin=5pt}
équivalent à
\geometry{hmargin=5pt, vmargin=5pt}
équivalent à
\geometry{margin={5pt,5pt}}
%%%% fin exemple %%%%

Exemple 2 :
\documentclass{report}
\usepackage{french}
\usepackage{geometry}
\geometry{scale=1.0, nohead}
\begin{document}
Ainsi, le texte apparaît beaucoup plus haut dans une page.
\end{document}
%%%% fin exemple %%%%

Exemple 3 :
\documentclass{report}
\usepackage{french}
\begin{document}
Voici une page normale pour comparer.
\end{document}
%%%% fin exemple %%%%

* Le package 'truncate' disponible sur CTAN dans
/macros/latex/contrib/other/misc/ permet de fixer la largeur
d'un texte.

* Il existe également le package 'typearea' disponible sur CTAN
dans /macros/latex209/contrib/script/.

* L'environnement narrow, de K. Reckdahl, ci-dessous permet
d'encapsuler des paragraphes de largeurs différentes.
%%%% debut macro %%%%
%-----------------------------------------------------------------
% \begin{narrow}{1.0in}{0.5in} produces text which is narrowed
% by 1.0 on left margin and 0.5 inches on right margin
% \begin{narrow}{-1.0in}{-0.5in} produces text which is widened
% by 1.0 on left margin and 0.5 inches on right margin
% Narrow environments can be nested and are ended by \end{narrow}
%-----------------------------------------------------------------
\newenvironment{narrow}[2]{%
\begin{list}{}{%
\setlength{\topsep}{0pt}%
\setlength{\leftmargin}{#1}%
\setlength{\rightmargin}{#2}%
\setlength{\listparindent}{\parindent}%
\setlength{\itemindent}{\parindent}%
\setlength{\parsep}{\parskip}%
}%
\item[]}{\end{list}}
%%%% fin macro %%%%

# 6.8 # Comment changer l'orientation d'un document ?
-----------------------------------------------------
* Globalement, pour passer en orientation portrait, il suffit de
mettre l'option landscape dans \documentstyle (LaTeX2.09) ou
dans \documentclass (LaTeX2e).

Exemple :
\documentclass[landscape]{report}
\usepackage{french}
\begin{document}
Voici un document écrit dans un sens non conventionnel.
\end{document}
%%%% fin exemple %%%%

* Le package 'lscape' de D. Carlisle (LaTeX2e), disponible sur
CTAN dans /macros/latex/packages/graphics/, permet de changer
localement d'orientation portrait vers paysage et vice versa. Il
définit l'environnement landscape.

Exemple :
\documentclass[11pt]{report}
\usepackage{lscape}

\begin{document}

\begin{landscape}
Un petit tour à la campagne,~\ldots
\end{landscape}

et nous voici de retour dans la galerie, après un changement de
page bien évidemment.

\end{document}
%%%% fin exemple %%%%

* Il existe également le package 'rotating' disponible sur CTAN
dans /macros/latex/contrib/supported/rotating/.

Exemple (tiré de la doc) :
\newcount\wang
\newsavebox{\wangtext}
\newdimen\wangspace
\def\wheel#1{\savebox{\wangtext}{#1}%
\wangspace\wd\wangtext
\advance\wangspace by 1cm%
\centerline{%
\rule{0pt}{\wangspace}%
\rule[-\wangspace]{0pt}{\wangspace}%
\wang=-180\loop\ifnum\wang<180
\rlap{\begin{rotate}{\the\wang}%
\rule{1cm}{0pt}#1\end{rotate}}%
\advance\wang by 10 \repeat}}
\wheel{Save the whale}
%%%% fin exemple %%%%

* Il existe aussi le programme docstrip disponible par ftp à
ftp.cdrom.com dans /macros/latex/unpacked/docstrip.tex.

# 6.9 # Comment justifier verticalement un paragraphe ?
-------------------------------------------------------
L'environnement vcenterpage ci-dessous permet de centrer
verticalement un texte sur une page seule.

%%%% debut macro %%%%
\newenvironment{vcenterpage}
{\newpage\vspace*{\fill}}
{\vspace*{\fill}\par\pagebreak}
%%%% fin macro %%%%

Exemple :
\begin{vcenterpage}
Texte qui apparait au milieu de la page.
\end{vcenterpage}
%%%% fin exemple %%%%

# 6.10 # Comment modifier la commande \caption ?
------------------------------------------------
* Le package 'hangcaption', disponible sur CTAN dans
/macros/latex209/contrib/misc/, dans lequel la commande
\isucaption remplace la commande \caption, permet de modifier la
mise en page de cette dernière. En particulier, il permet de
définir \captionwidth.

* Il existe également les packages 'caption' et 'caption2'
disponibles sur CTAN dans
/macros/latex/contrib/supported/caption/ qui permettent de
modifier la commande \caption classique (largeur, style,
fonte, ...).

Exemples :
\usepackage[small,hang]{caption2}
\renewcommand{\captionfont}{\it \small}
\renewcommand{\captionlabelfont}{\it \bf \small}
\renewcommand{\captionlabeldelim}{ :}
%%%% fin exemple %%%%

Remarque : il est recommandé d'utiliser 'caption2' plutôt que
'caption'.

* Le package 'topcapt' disponible sur CTAN dans
/macros/latex/contrib/misc/ définit la commande \topcaption qui
permet de placer le texte de la commande caption au dessus du
flottant auquel elle est rattachée.

* Le package 'french' de B. Gaulle propose la commande
\unnumberedcaptions qui permet de supprimer la numérotation des
flottants.

* Pour avoir une présentation du type :
Figure 1. Voici le texte de la légende d'une figure. Mais
lorsque la légende est longue, elle se présente
comme ceci.

au lieu de :
Figure 1. Voici le texte de la légende d'une figure. Mais
lorsque la légende est longue, elle se présente comme
ceci.

on peut utiliser \caption[texte]{\protect\parbox[t]{wd}{texte}}
où wd est la largeur de la boîte (cette largeur peut être adaptée
à la taille de la figure automatiquement si la figure est
elle-même dans une boite dont on récupère la largeur). \protect
est absolument nécessaire pour éviter une erreur. Le texte au
début [texte] est celui qui apparaîtra dans la table des figures
(il peut éventuellement être plus court).

Plus simplement, le package 'caption2' permet de faire la même
chose avec l'option hang.

* La variable \figurename permet de redefinir le nom du titre de
la figure. De même pour \tablename et les tableaux.

Exemple :
\def\figurename{Croquis}
%%%% fin exemple %%%%

# 6.11 # Comment modifier un changement de page ?
-------------------------------------------------
* \newpage impose un changement brutal de page.

* \pagebreak[n] où n représente l'autorité avec laquelle on veut
changer de page (n compris entre 1 et 5). Cette commande garde
la justification, 1 impose une contrainte faible, 5 impose une
contrainte sévère.

* \nopagebreak[n] idem pour empêcher une coupure de page.

* \clearpage agit comme un \newpage mais libère le buffer de
flottants.

* \cleardoublepage fait la même chose et force un redémmarrage
sur une page impaire.

* \enlargethispage{lgr} impose à LaTeX2e de comprimer (lgr
négatif) ou d'expanser (lgr positif) le contenu d'une page.
Ceci pour éviter que la page suivante contienne trop peu de
texte.

Exemples :
\enlargethispage{1cm} % ajoute un cm
\enlargethispage{-2\baselineskip} % supprime deux lignes dans la
% page
%%%% fin exemple %%%%

Remarque : \enlargethispage*{lgr} donne une autorité plus grande
à la commande \enlargethispage en l'obligeant à agir
sur les espacements élastiques verticaux contenus
dans la page courante.

* J.P. Drucbert donne ci-dessous un petit papier décrivant un
style block.sty bricolé à partir de macro de diverses origines
(dont D. Arseneau, assez connu). Si vous passez latex sur ce
papier, block.sty sera créé. Les macros les plus intéressantes
sont \need{dim} et \lneed{N}, qui provoquent un saut de page
s'il reste moins que la dimension dim (ou N fois \baselinskip)
verticalement sur la page. Ce n'est pas parfait, mais cela m'a
rendu des services.

%%%% debut macro %%%%
\begin{filecontents}{block.sty}
\ifx\endBlock\undefined
\def\block{\begingroup%
\def\endblock{\egroup\endgroup}%
\vbox\bgroup}%
\long\def\Block{\begingroup%
\def\endBlock{\unskip\egroup\endgroup}%
\pagebreak[2]\vspace*{\parskip}\vbox\bgroup%
\par\noindent\ignorespaces}
\long\def\IBlock{\begingroup%
\def\endIBlock{\unskip\egroup\endgroup}%
\pagebreak[2]\vspace*{\parskip}\vbox\bgroup\par\ignorespaces}
\def\need#1{\ifhmode\unskip\par\fi \penalty-100 \begingroup
% preserve \dimen@, \dimen@i
\ifdim\prevdepth>\maxdepth \dimen@i\maxdepth
\else \dimen@i\prevdepth\fi
\kern-\dimen@i
\dimen@\pagegoal \advance\dimen@-\pagetotal % space left
\ifdim #1>\dimen@
\vfill\eject\typeout{WARNING- EJECT BY NEED}
\fi
\kern\dimen@i
\endgroup}
\def\lneed#1{\need{#1\baselineskip}}
% \begin{block} ... \end{block} delimite un bloc qui restera,
% si possible, sur une seule page.
\long\def\TBlock{\begingroup%
\def\endTBlock{\unskip\egroup\endgroup}%
\pagebreak[2]\vspace*{\parskip}\vtop\bgroup%
\par\noindent\ignorespaces}
\else
\typeout{block.sty already loaded}
\fi
\endinput

\def\need#1{\par \penalty-100 \begingroup
% preserve \dimen@, \dimen@i
\ifdim\prevdepth>\maxdepth \dimen@i\maxdepth
\else \dimen@i\prevdepth\fi
\kern-\dimen@i
\dimen@\pagegoal \advance\dimen@-\pagetotal % space left
\ifdim #1>\dimen@ \vfil \eject \fi
\kern\dimen@i
\endgroup}
\end{filecontents}

\documentclass[12pt,a4paper]{article}
\def\bs{\texttt{\char'134}}
\parskip=12pt plus1pt minus0.5pt
\usepackage[english,francais]{babel}
\usepackage[T1]{fontenc}
\usepackage[isolatin]{inputenc}
\begin{document}
\clearpage
\section{Paquetage \texttt{block}}\label{BLOCK+}
Ce paquetage offre divers outils permettant d'empêcher qu'une
rupture de page se produise à un certain endroit.

\subsection{Blocs Protégés}

Vous pouvez protéger une zone contre la rupture de page. Il
suffit pour cela de la placer dans un environnement
\texttt{Block} ou dans un environnement \texttt{IBlock}. Dans le
premier cas (\texttt{Block}), le premier paragraphe se trouvant
dans la zone ne sera pas indenté, mais le sera dans le second
cas (\texttt{IBlock}). Ceci est utile en particulier pour
éviter de séparer un texte et un exemple qui l'accompagne. Les blocs ainsi
protégés doivent, bien entendu, rester assez petits. La syntaxe
est (vous pouvez remplacer \texttt{Block} par \texttt{IBlock}):
\begin{quote}\tt
%\begin{tabular}{l}
\bs begin\{Block\}\\
\qquad\ldots\\
zone protégée\\
\qquad\ldots\\
\bs end\{Block\}
%\end{tabular}
\end{quote}

Cette méthode, très simple, a l'inconvénient de ne pas pouvoir
s'appliquer lorsque la zone à protéger doit contenir une
commande de sectionnement (c'est à dire du même type que
\verb|\section|), une note en bas de page, une note marginale ou
un corps mobile (figure ou table). Dans ce cas, il faudra
utiliser une des commandes du paragraphe suivant.

\subsection{Réservations Verticales}
Vous pouvez aussi demander de changer de page (ou de colonne, si votre
document est sur deux colonnes) s'il ne reste pas verticalement assez
de place sur la page. Deux commandes de réservation verticale sont
disponibles: \verb|\need{|{\em dimension\/}\verb|}|, dont le paramètre est une
longueur, et \verb|\lneed{|{\em nombre\/}\verb|}|, dont le paramètre est
le nombre de lignes équivalant à l'espace vertical demandé (avec cette forme
le paramètre est plus facile à estimer).

\end{document}
%%%% fin macro %%%%

* Pour éviter les coupures de page (ou les coupures de colonnes
avec multicol.sty) à l'intérieur des \item dans une description
(et d'ailleurs dans toute liste), solution simple sortie du
TeXbook:
\begin{description}\interlinepenalty 10000

# 6.12 # Comment obtenir des colonnes parallèles ?
--------------------------------------------------
Le package 'parallel' disponible sur CTAN dans
/macros/latex/contrib/supported/parallel/ permet d'obtenir deux
colonnes dont l'une peut contenir la traduction de l'autre.

# 6.13 # Comment définir l'espace de début de paragraphe ?
----------------------------------------------------------
* Il faut valoriser la variable \parindent.

Exemple :
\setlength{\parindent}{1cm}
%%%% fin exemple %%%%

* Pour agir localement, on peut utiliser \hspace*{lgr}.

* Le package 'indentfirst' permet de forcer LaTeX à indenter le
premier paragraphe après une nouvelle section (indentation
normalement non utilisée en typographie française).

# 6.14 # Comment supprimer la date sur une page de titre ?
----------------------------------------------------------
Il faut ajouter la commande \date{} dans le préambule du
document.

# 6.15 # Comment mettre en valeur la première lettre d'un
---------------------------------------------------------
chapitre ?
----------
* Il faut utiliser le package 'dropcaps' de F. Lauwers.
Il est disponible sur CTAN dans
/macros/latex209/contrib/dropcaps/. Ce package est utilisable
avec LaTeX2.09 et LaTeX2e.

* Les packages 'initial' et 'initials' pour LaTeX2e sont
disponibles sur CTAN. Le second que l'on peut trouver dans
/fonts/gothic/yinit/ fait appel à des fontes yinit particulières.

* Il existe également un package 'drop' pour LaTeX2.09, mais
compatible LaTeX2e, disponible sur CTAN dans
/macros/latex209/contrib/misc/.

Exemple :
\documentclass[12pt,a4paper]{article}
\usepackage{drop}
\font\largefont=yinitas % fontes yinit
begin{document}
\drop{D}OES THERE EXIST a field with 4 elements?
\end{document}
%%%% fin exemple %%%%

* 'bigdrop' accessible sur CTAN dans/CTAN/digests/ttn est une
macro TeX compatible LaTeX.

* Le package 'bigstart' pour LaTeX2.09 et LaTeX2e permet
également de faire cela.

* Il existe également le package 'picinpar' pour LaTeX2.09
disponible sur CTAN dans /macros/latex209/contrib/picinpar/ et
dans /systems/msdos/4alltex/disk04/.

* On peut également définir sa propre macro :
%%%% debut macro %%%%
\font\capfont=cmbx12 at 24.87 pt % or yinit, or...?
\newbox\capbox \newcount\capl \def\a{A}
\def\docappar{\medbreak\noindent\setbox\capbox\hbox{%
\capfont\a\hskip0.15em}\hangindent=\wd\capbox%
\capl=\ht\capbox\divide\capl by\baselineskip\advance\capl by1%
\hangafter=-\capl%
\hbox{\vbox to8pt{\hbox to0pt{\hss\box\capbox}\vss}}}
\def\cappar{\afterassignment\docappar\noexpand\let\a }
%%%% fin macro %%%%

Exemple :
\cappar Il était une fois un petit chaperon rouge qui avait une
grand-mère qui habitait de l'autre côté de la foret. Un jour,
alors que sa grand-mère était malade, le petit chaperon rouge
décida de lui rendre visite~\ldots
%%%% fin exemple %%%%

* Un petit dernier, le package 'dropping' qui étend le
package 'dropcaps' est disponible sur CTAN dans
/macros/latex/contrib/other/dropping/.

Exemple :
\dropping{3}{\itshape{} Voici} un exemple de ce que permet de
faire le magnifique package dropping de M. Dahlgren. La commande
\texttt{\\dropping} peut prendre en argument un mot comme c'est le cas ici
ou une simple lettre.
%%%% fin exemple %%%%

* Le package 'french' de B. Gaulle propose également la commande
\lettrine.

Exemple :
\lettrine{UN jour,} mon prince~\ldots
%%%% fin exemple %%%%

# 6.16 # Comment préserver les tabulations en mode verbatim ?
-------------------------------------------------------------
* Le package 'moreverb' est disponible sur CTAN dans
/macros/latex/contrib/other/misc/. Il propose notamment un
environnement verbatimtab qui permet de conserver des
tabulations.

Exemple :
\begin{verbatimtab}
int pattern(char *p, int n, int m)
{
int orig = current_position();
int new_pos;

goto_line(n);

if(p && forward_search(p) && (current_line()<m)){
new_pos = current_position();
goto_char(orig);
return(new_pos);
}
return(-1);
}
\end{verbatimtab}
%%%% fin exemple %%%%

* Il existe également le package 'verbasef' (verbatim automatic
segmentation of external files) disponible sur CTAN. Il utilise
l'environnement figure.

* On peut également inclure les lignes suivantes dans le
préambule du document :
%%%% debut macro %%%%
\makeatletter
{\catcode`\^^I=\active
\gdef\verbatim{\catcode`\^^I=\active\def^^I{\hspace*{4em}}%
\@verbatim \frenchspacing\@vobeyspaces \@xverbatim}}
\makeatother
%%%% fin macro %%%%

# 6.17 # Comment modifier les entêtes de chapitre ?
---------------------------------------------------
Il faut modifier la macro makechapterhead.

Ajouter, par exemple, dans le preambule :
%%%% debut macro %%%%
\makeatletter
\def\@makechapterhead#1{%
\vspace*{50\p@}%
{\parindent \z@ \raggedright \normalfont
\interlinepenalty\@M
\ifnum \c@secnumdepth >\m@ne
\Huge\bfseries \thechapter\quad
\fi
\Huge \bfseries #1\par\nobreak
\vskip 40\p@
}}

\def\@makeschapterhead#1{%
\vspace*{50\p@}%
{\parindent \z@ \raggedright
\normalfont
\interlinepenalty\@M
\Huge \bfseries #1\par\nobreak
\vskip 40\p@
}}
\makeatother
%%%% fin macro %%%%

# 6.18 # Comment définir des tabulations ?
-----------------------------------------
Il faut utiliser l'environnement tabbing qui permet de placer
des marques d'alignement dans un texte.

Exemple :
\begin{tabbing}
Voici \= des \= marques \= de tabulation \\
\> la je m'aligne sur la première \\
\> \> \> la sur la troisième \\
\hspace{3cm} \= \hspace{2cm} \= \kill
un \> autre \> exemple.
\end{tabbing}
%%%% fin exemple %%%%

# 6.19 # Comment obtenir des lettres accentuées dans tabbing ?
--------------------------------------------------------------
* Pour produire un \'{e} dans un environnement tabbing ou la
commande \' a été redéfinie, il faut utiliser \a'{e} ou \a'e

* Le style suivant de J.P. Drucbert permet de remplacer
l'environnement standard tabbing par l'environnement Tabbing
dans lequel les commandes \` \' \> \< \= \+ \- sont remplacées
par \TAB` \TAB' \TAB> \TAB< \TAB= \TAB+ \TAB- ce qui permet de
préserver les commandes d'accent (\' \` ou \=).

%%%% debut macro %%%%
\ProvidesPackage{Tabbing}[1996/01/16]
\NeedsTeXFormat{LaTeX2e}[1995/12/01]
\gdef\Tabbing{\lineskip \z@skip
% \let\>\@rtab
% \let\<\@ltab
% \let\=\@settab
% \let\+\@tabplus
% \let\-\@tabminus
% \let\`\@tabrj
% \let\'\@tablab
\def\TAB##1{\ifx ##1>\@rtab\else
\ifx ##1<\@ltab\else
\ifx ##1=\@settab\else
\ifx ##1+\@tabplus\else
\ifx ##1-\@tabminus\else
\ifx ##1`\@tabrj\else
\ifx ##1'\@tablab\else
\PackageError{Tabbing}%
{Bad argument ##1 for Tabbing
specification} \fi\fi\fi\fi\fi\fi\fi}
\let\\=\@tabcr
\global\@hightab\@firsttab
\global\@nxttabmar\@firsttab
\dimen\@firsttab\@totalleftmargin
\global\@tabpush\z@ \global\@rjfieldfalse
\trivlist \item\relax
\if@minipage\else\vskip\parskip\fi
\setbox\@tabfbox\hbox{\rlap{\indent\hskip\@totalleftmargin
\the\everypar}}\def\@itemfudge{\box\@tabfbox}%
\@startline\ignorespaces}
\gdef\endTabbing{%
\@stopline\ifnum\@tabpush >\z@ \@badpoptabs \fi\endtrivlist}
\endinput
%%%% fin macro %%%%

# 6.20 # Comment encadrer du texte ?
------------------------------------
* Une solution consiste à définir un tableau d'une seule
cellule.

* On peut aussi utiliser :
\fbox{
\begin{minipage}{0.7\textwidth}
Texte...
\end{minipage}
}

* On peut également se définir son propre environnement.

Exemple :
\newsavebox{\fmbox}
\newenvironment{fmpage}[1]
{\begin{lrbox}{\fmbox}\begin{minipage}{#1}}
{\end{minipage}\end{lrbox}\fbox{\usebox{\fmbox}}}

Utilisation :
\begin{fmpage}{3cm}
Texte à encadrer dans une boite ne dépassant pas 3
centimètres de large.
\end{fmpage}
%%%% fin exemple %%%%

* Il existe également le package 'fancybox', disponible sur CTAN
dans /macros/latex/contrib/others/seminar/inputs/, qui définit
des fonctions telles que \shadowbox, \doublebox, \ovalbox,...

Exemple :
\shadowbox{Texte ombré.}
\doublebox{Texte doublement encadré.}
\ovalbox{Texte dans un cadre aux coins arrondis.}
%%%% fin exemple %%%%

* Le package 'boxedminipage' est un vieux style LaTeX2.09
disponible sur CTAN dans /macros/latex209/contrib/misc/.

* Voir également le package 'hh' disponible sur CTAN dans
/macros/latex/contrib/supported/hh/.

* Le package 'niceframe' disponible sur CTAN dans
/macros/latex/contrib/supported/niceframe/ permet de définir des
cadres pleine page.

* Pour encadrer un texte pouvant s'étendre sur plusieurs pages,
il existe le package 'eclbkbox' :
%%%% debut macro %%%%
% eclbkbox.sty by Hideki Isozaki, 1992
% Date: May 28, 1993

\newbox\bk@bxb
\newbox\bk@bxa
\newif\if@bkcont
\newif\ifbkcount
\newcount\bk@lcnt

\def\breakboxskip{2pt}
\def\breakboxparindent{1.8em}

\def\breakbox{\vskip\breakboxskip\relax
\setbox\bk@bxb\vbox\bgroup
\advance\linewidth -2\fboxrule
\advance\linewidth -2\fboxsep
\hsize\linewidth\@parboxrestore
\parindent\breakboxparindent\relax}

% \@tempdimb: amount of vertical skip
% between the first line (\bk@bxa) and the rest (\bk@bxb)
\def\bk@split{%
\@tempdimb\ht\bk@bxb % height of original box
\advance\@tempdimb\dp\bk@bxb
\setbox\bk@bxa\vsplit\bk@bxb to\z@ % split it
\setbox\bk@bxa\vbox{\unvbox\bk@bxa}% recover height & depth of \bk@bxa
\setbox\@tempboxa\vbox{\copy\bk@bxa\copy\bk@bxb}% naive concatenation
\advance\@tempdimb-\ht\@tempboxa
\advance\@tempdimb-\dp\@tempboxa}% gap between two boxes

% \@tempdima: height of the first line (\bk@bxa) + fboxsep
\def\bk@addfsepht{%
\setbox\bk@bxa\vbox{\vskip\fboxsep\box\bk@bxa}}

\def\bk@addskipht{%
\setbox\bk@bxa\vbox{\vskip\@tempdimb\box\bk@bxa}}

% \@tempdima: depth of the first line (\bk@bxa) + fboxsep
\def\bk@addfsepdp{%
\@tempdima\dp\bk@bxa
\advance\@tempdima\fboxsep
\dp\bk@bxa\@tempdima}

% \@tempdima: depth of the first line (\bk@bxa) + vertical skip
\def\bk@addskipdp{%
\@tempdima\dp\bk@bxa
\advance\@tempdima\@tempdimb
\dp\bk@bxa\@tempdima}

\def\bk@line{%
\hbox to \linewidth{\ifbkcount\smash{\llap{\the\bk@lcnt\ }}\fi
\vrule \@width\fboxrule\hskip\fboxsep
\box\bk@bxa\hfil
\hskip\fboxsep\vrule \@width\fboxrule}}

\def\endbreakbox{\egroup
\ifhmode\par\fi{\noindent\bk@lcnt\@ne
\@bkconttrue\baselineskip\z@\lineskiplimit\z@
\lineskip\z@\vfuzz\maxdimen
\bk@split\bk@addfsepht\bk@addskipdp
\ifvoid\bk@bxb % Only one line
\def\bk@fstln{\bk@addfsepdp
\vbox{\hrule\@height\fboxrule\bk@line\hrule\@height\fboxrule}}%
\else % More than one line
\def\bk@fstln{\vbox{\hrule\@height\fboxrule\bk@line}\hfil
\advance\bk@lcnt\@ne
\loop
\bk@split\bk@addskipdp\leavevmode
\ifvoid\bk@bxb % The last line
\@bkcontfalse\bk@addfsepdp
\vtop{\bk@line\hrule\@height\fboxrule}%
\else % 2,...,(n-1)
\bk@line
\fi
\hfil\advance\bk@lcnt\@ne
\if@bkcont\repeat}%
\fi
\leavevmode\bk@fstln\par}\vskip\breakboxskip\relax}

\bkcountfalse
%%%% fin macro %%%%

L'encadrement du texte est obtenu par l'environnement breakbox.

\bkcounttrue : les lignes sont numérotées.
\bkcountfalse : elles ne le sont pas (défaut).

On peut emboîter des environnements breakbox.

# 6.21 # Comment gérer des URL WWW ?
------------------------------------
* Il existe le package 'url' disponible sur CTAN dans
/macros/latex/contrib/other/misc/ qui permet de gérer les
coupures des URL WWW un peu longues. Il est également capable de
gérer les adresses e-mail, les liens hypertexte, les noms de
directories. En outre, cet outil est paramètrable.

Exemple :
\usepackage{url}
\urlstyle{sf}
..
\url{http:\\hostname\~username}
%%%% fin exemple %%%%

* Le package 'path' disponible sur CTAN dans /macros/eplain/
offre les mêmes possibilités.

* La macro \discretionary permet également de dire comment
couper une chaîne de caractères :
\discretionary{Avant la coupure}{après}{s'il n'y en a pas}

Remarque : \- est défini comme \discretionary {-}{}{}

# 6.22 # Comment mettre en page des exercices dont les solutions
----------------------------------------------------------------
sont reportées dans un autre paragraphe ?
-----------------------------------------
L'objectif ici est de pouvoir saisir dans le fichier source les
textes des exercices suivis de leurs solutions, alors qu'au
niveau de la mise en page du document, les solutions
apparaîssent groupées dans un autre paragraphe/chapitre.

* Le package 'answers', de M. Piff, disponible sur CTAN dans
macros/latex/contrib/supported/answers/ permet également de
réaliser ce genre d'exercice. Il permet entre autres :
+ d'avoir plusieurs types de solutions (ex: réponse numérique
seule ou détail),
+ d'inclure les solutions (l'une, l'autre ou les deux dans
le cas précité) dans le texte (après une marque spécifique
si désiré),
+ mettre les solutions à la fin,
+ ne pas mettre les solutions.

Exemple :
%%
%% This is file `ansexam2.tex',
%% generated with the docstrip utility.
%%
%% The original source files were:
%%
%% answers.dtx (with options: `ex2')
%%
\documentclass[12pt,a4paper]{article}
\usepackage{answers}%\usepackage[nosolutionfiles]{answers}
% def d'un environnement Exercise numerote
\newtheorem{Exc}{Exercise}
\newenvironment{Ex}{\begin{Exc}\normalfont}{\end{Exc}}
% Trois types de solutions sont proposes
\Newassociation{solution}{Soln}{test}
\Newassociation{hint}{Hint}{test}
\Newassociation{Solution}{sSol}{testtwo}
\newcommand{\prehint}{~[Hint]}
\newcommand{\presolution}{~[Solution]}
\newcommand{\preSolution}{~[Homework]}
% test
\newcommand{\Opentesthook}[2]%
{\Writetofile{#1}{\protect\section{#1: #2}}}
% introduction de la solution
\renewcommand{\Solnlabel}[1]{\emph{Solution #1}}
\renewcommand{\Hintlabel}[1]{\emph{Hint #1}}
\renewcommand{\sSollabel}[1]{\emph{Solution to #1}}

\begin{document}
% gestion des fichiers contenant les solutions
\Opensolutionfile{test}[ans2]{Solutions}
\Writetofile{test}{\protect\subsection{Some Solutions}}
\Opensolutionfile{testtwo}[ans2x]
\Writetofile{testtwo}{%
\protect\subsection{Extra Solutions}}

% Exercices
\section{Exercises}
\begin{Ex}
An exercise with a solution.
\begin{solution}
This is a solution.
\relax{}
\end{solution}
\end{Ex}
\begin{Ex}
An exercise with a hint and a secret solution.
\begin{hint}
This is a hint.
\end{hint}
\begin{Solution}
This is a secret solution.
\end{Solution}
\end{Ex}
\begin{Ex}
An exercise with a hint.
\begin{hint}
This is a hint.
\end{hint}
\end{Ex}
% gestion des fichiers contenant les solutions
\Closesolutionfile{test}
\Readsolutionfile{test}
% \clearpage
\hrule
\Closesolutionfile{testtwo}
\Readsolutionfile{testtwo}
\end{document}
%%
%% End of file `ansexam2.tex'.

%%%% fin exemple %%%%

* On peut également trouver le package 'exam' sur CTAN dans
/macros/latex/contrib/supported/exams/.

# 6.23 # Comment positionner un objet dans une page ?
-----------------------------------------------------
* Pour pouvoir positionner un objet aux coordonnées x,y par
rapport au coin supérieur gauche d'une page, il suffit
d'utiliser le package 'atxy' disponible par ftp à
ftp.univ-orleans.fr dans le module l209misc.zip du répertoire
/pub/tex/PC/AsTeX/Paq_Base/.

Exemple :
\atxy(3cm,2.5cm) {toto adresse toto}
\atxy(3cm,4cm) {date}
%%%% fin exemple %%%%

Remarque : petit défaut, si le document ne contient que des
commandes \atxy il n'y a pas de dvi généré. Il faut
donc ajouter n'importe quoi au début du document, ~
par exemple, pour que ça marche.

# 6.24 # Comment lier le placement des flottants aux sections ?
---------------------------------------------------------------
Pour lier la position des éléments flottants aux limites
de sections, D. Arseneau a développé le package 'placeins'
disponible sur CTAN dans
/macros/latex/contrib/other/misc. Ce package définit la
commande \FloatBarrier qui force le placement des flottants
avant son appel.

# 6.25 # Comment griser le fond (background) d'un paragraphe ?
--------------------------------------------------------------
* Le package 'psboxit', disponible sur CTAN dans
/macros/latex2.09/contrib/misc/ et en particulier
l'environnement "boxitpara" permet de faire cela.

* Le package 'shadbox' disponible sur CTAN dans
/macros/latex/contrib/other/shadbox permet de griser toute
boîte, texte, figure, ....

* Le package 'shading' disponible sur CTAN dans
/macros/latex209/contrib/shading/ permet de griser un
paragraphe.

* De même, le package 'shade', de P. Schmitt, est disponible sur
CTAN dans /macros/generic/

* On peut également utiliser le package 'color'.

Exemple :
\colorbox[gray]{0.5}{some words}
%%%% fin exemple %%%%

# 6.26 # Comment modifier l'espace inter-colonnes ?
---------------------------------------------------
Il faut modifier la variable \columnsep.

Exemple :
\addtolength{\columnsep}{5mm}
%%%% fin exemple %%%%

# 6.27 # Comment modifier les environnements de liste ?
-------------------------------------------------------
* L'environnement list permet de définir son propre style de
liste. Sa syntaxe est la suivante :
\begin{list}{label}{mep}\end{list}
+ l'argument label permet de définir le symbole qui sera
associé à chaque élément de la liste.
+ mep permet de définir la mise en page des éléments de la
liste. Les paramètres utilisés pour définir cette mise en page
sont les suivants :
\topsep espace vertical supplémentaire (ajoute à \parskip)
inséré entre le texte précédent la liste et le 1er objet
de la liste
\partosep espace vertical supplémentaire inséré devant la liste
si celle-ci est précédée d'une ligne blanche
\itemsep espace vertical supplémentaire (ajouté à \parsep)
inséré entre les éléments d'une liste.

Exemple :
\newenvironment{maliste}%
{ \begin{list}%
{$\bullet$}%
{\setlength{\labelwidth}{30pt}%
\setlength{\leftmargin}{35pt}%
\setlength{\itemsep}{\parsep}}}%
{ \end{list} }

Utilisation :
\begin{maliste}
\item premier élément
\item deuxième élément
\begin{maliste}
\item petit 1
\item petit 2
\end{maliste}
\end{maliste}
%%%% fin exemple %%%%

* Le petit bout de code ci-dessous, de M. Boyer
(mbo...@robot.ireq.ca), définit les commandes :
+ \noitemsep pour supprimer tout espacement vertical entre les
items des environnements \itemize, \enumerate et \description.
+ \doitemsep pour les remettre.
Pour l'utiliser, il suffit de le sauvegarder dans un fichier
.sty et de l'inclure dans son document par une commande
\usepackage.

%%%% debut macro %%%%
%% ----------------------------------------------------
%% Copyright (c) 1993 Hydro-Quebec mbo...@robot.ireq.ca
%% ----------------------------------------------------

%% Bring items closer together in list environments
% Prevent infinite loops
\let\orig@Itemize =\itemize
\let\orig@Enumerate =\enumerate
\let\orig@Description =\description
% Zero the vertical spacing parameters
\def\Nospacing{\itemsep=0pt\topsep=0pt\partopsep=0pt%
\parskip=0pt\parsep=0pt}
% Redefinition de art12.sty pour commencer a la marge de gauche
%\leftmargini 1.2em % 2.5em

\def\noitemsep{
% Redefine the environments in terms of the original values
\renewenvironment{itemize}{\orig@Itemize\Nospacing}{\endlist}
\renewenvironment{enumerate}{\orig@Enumerate\Nospacing}{\endlist}
\renewenvironment{description}{\orig@Description\Nospacing}%
{\endlist}
}

\def\doitemsep{
% Redefine the environments to the original values
\renewenvironment{itemize}{\orig@Itemize}{\endlist}
\renewenvironment{enumerate}{\orig@Enumerate}{\endlist}
\renewenvironment{description}{\orig@Description}{\endlist}
}
%%%% fin macro %%%%

* La macro ci-dessous de T. Murphy permet de remplacer les
numéros de l'environnement enumerate par des caractères grecs :
%%%% debut macro %%%%
\makeatletter
\def\greek#1{\expandafter\@greek\csname c@#1\endcsname}
\def\@greek#1{\ifcase#1\or$\alpha$\or$\beta$\fi}% as many as you
% need
\renewcommand{\theenumi}{\greek{enumi}}
\makeatother
%%%% fin macro %%%%

* Les définitions suivantes :
\renewcommand{\labelitemi}{\textbullet}
\renewcommand{\labelitemii}{---}
\renewcommand{\labelitemiii}{votre-label-pour-le-niveau-iii}
\renewcommand{\labelitemiv}{votre-label-pour-le-niveau-iv}
permettent de redéfinir les caractères utilisés par
l'environnement itemize pour ces différents niveaux
d'encapsulation.

Remarque : avec certains packages comme french, il faut placer
ces nouvelles définitions de commande après le
\begin{document}.

De même, utilisez \descriptionlabel pour changer le style des
étiquettes de l'environment description.

Exemple :
\renewcommand\descriptionlabel[1]{\hspace\labelsep\normalfont%
\itshape #1:}
produit des étiquettes en italique, avec deux points :
\begin{description}
\item[Carte maîtresse] As
\item[Carte maîtresse à l'atout] Valet
\end{description}
%%%% fin exemple %%%%

* Pour réduire globalement l'espace entre les items d'une liste
on peut également utiliser le bout de code suivant de M. Wooding
%%%% debut macro %%%%
\makeatletter
\toks@\expandafter{\@listI}
\edef\@listI{\the\toks@\setlength{\parsep}{1pt}}
\makeatother
%%%% fin macro %%%%

* Le package 'mdwlist', de M. Wooding, disponible sur CTAN dans
/macros/latex/contrib/supported/mdwtools/ permet de redéfinir
certains paramètres de mise en page des listes qui ne sont pas
facile d'accès sous LaTeX.

# 6.28 # Comment souligner plusieurs lignes ?
---------------------------------------------
Pour souligner un texte qui comprend des retours à la ligne, il
faut utiliser le package 'ulem' disponible sur CTAN dans
/macros/latex/contrib/other/misc/. Ce package redéfinit en fait
le mode emphasize. Les commandes \normalem et \ULforem
permettent de passer du mode \emph classique au mode \emph
souligné. En mode souligné, la commande devient paramétrable
pour changer le style du souligné ou biffer des mots.

Exemple :
\documentclass{report}
\usepackage{french}
\usepackage[normalem]{ulem}
\pagestyle{empty}
\begin{document}

\normalem
Voici le mode "\emph{emphasize}" classique.

\ULforem
Voici le mode "\emph{emphasize}" souligné. \emph{Il permet
également de gérer les retours à la ligne tout en restant dans
le style emphasize.}

Les autres possibilités sont les suivantes~:
\begin{itemize}
\item vagues~: \uwave{texte}
\item barré~: \sout{texte}
\item rayé~: \xout{texte}
\end{itemize}

\end{document}
%%%% fin exemple %%%%

Remarque : \underline ne permet pas de gérer les retours à la
ligne du fait qu'il encapsule le texte passé en
argument dans une boîte.

# 6.29 # Comment réaliser des onglets ?
---------------------------------------
* Pour insérer un carré noir, décalé vers le bas à chaque
nouveau chapitre, le long de la marge des pages de droite d'un
document, on peut utiliser le package 'fancyhdr' ou
'fancyheadings'. Ces packages sont disponibles sur CTAN
respectivement dans /macros/latex/contrib/supported/fancyhdr/ et
/macros/latex/contrib/other/.

Remarque : On note toutefois qu'il est peu probable qu'une
imprimante puisse accéder au ras de la marge. La
solution consiste alors à définir un format de
document plus petit et utiliser le massicot.
Attention dans ce cas lors de la définition des
marges.

* On peut également utiliser le package 'onglet' définit
ci-dessous par B. Bayart. Celui-ci nécessite le package
'everyshi' disponible sur CTAN dans
/macros/latex/contrib/supported/everyshi/.

%%%% debut macro %%%%
\ProvidesPackage{onglet}[1996/07/25 B. Bayart]
\RequirePackage{everyshi}

\newcounter{maxchapter}
\newcounter{tmpa}
\newlength{\basehauteur}
\setlength{\basehauteur}{1cm}
\newlength{\ajoutdroite}
\newlength{\htcclv}
\def\concatener{%
\setlength{\ajoutdroite}{\textheight}
\divide\ajoutdroite by \basehauteur
\setcounter{maxchapter}{\number\ajoutdroite}
\setcounter{tmpa}{\value{chapter}}
\addtocounter{tmpa}{-1}
\divide\value{tmpa} by\value{maxchapter}
\multiply\value{tmpa} by\value{maxchapter}
\advance\value{tmpa} by -\value{chapter}
\addtocounter{tmpa}{-1}
\setlength{\ajoutdroite}{\paperwidth}
\setlength{\htcclv}{\ht255}
\addtolength{\ajoutdroite}{-\wd255}
\addtolength{\ajoutdroite}{-1in}
\addtolength{\ajoutdroite}{-1.5cm}
\setbox255=\vbox{\hbox to \wd255{%
\box255%\relax
\rlap{\vbox to \htcclv{%
\vskip-\value{tmpa}\basehauteur
\hbox{%
\hskip\ajoutdroite\relax
\usebox{\laboite}%
}%
\vfill
}}%
\hfill}}%
}
\newsavebox{\laboite}
\def\faireboite{\sbox{\laboite}%
{\hbox to 1.5cm{\let\protect\relax
\huge\thechapter\hfill\vrule height 1em depth 0pt width 5mm}}}

\AtBeginDocument{\EveryShipout{\faireboite\concatener}}
\endinput
%%%% fin macro %%%%

# 6.30 # Comment réaliser des QCM ?
-----------------------------------
Il existe le package 'exam' disponible sur CTAN dans
/macros/latex/contrib/supported/exams/. Ce package permet entre
autres de paramétrer les questions de manière à ce que les
propositions apparaîssent dans un ordre aléatoire.

Exemple :
Combien le cheval possède t'il de pattes ?
\begin{choice}[\random]
\baditem{deux pattes}
\baditem{quatres pattes}
\baditem{zéro patte}
\end{choice}
%%%% fin exemple %%%%

# 6.31 # Comment modifier l'orientation des flottants ?
-------------------------------------------------------
* Le package 'rotating' disponible sur CTAN dans
/macros/latex/contrib/supported/rotating/ offre deux
environnements sidewaysfigure pour les figures et sidewaystable
pour les tableaux. Les figures ou tableaux sont alors insérés
sur une page séparée.

Exemple :
\begin{sidewaystable}
\begin{tabular}{|c|c|}
\hline
Un & Deux \\
Trois & Quatre \\
\hline
\end{tabular}
\end{sidewaystable}
%%%% fin exemple %%%%

* On peut également utiliser l'environnement sideways du
package 'rotating'. De la même manière cet environnement génère
une page séparée.

Exemple :
\begin{figure}
\begin{sideways}
\includegraphics{foobar.ps}
\end{sideways}
\end{figure}
%%%% fin exemple %%%%

Remarque : il est fréquent que les visualiseurs de fichiers dvi
ne sachent pas gérer les changements d'orientation à
l'intérieur d'un document. Il faut pour voir le
résultat définitif utiliser un visualiseur de
postscript.

# 6.32 # Comment faire référence plusieurs fois à la même note
--------------------------------------------------------------
de bas de page ?
----------------
* Il faut utiliser les commandes \footnotemark[] et
\footnotetext[]{}. \footnotemark permet de gérer le compteur de
notes et \footnotetext permet d'insérer le texte correspondant
en bas de page.

Exemple :
bla blabla\footnotemark[1] bla bla bla blablabla\footnotemark[2]
bla bla bla bla blabla\footnotemark[1] bla.

\footnotetext[1]{double bla}
\footnotetext[2]{triple bla}
%%%% fin exemple %%%%

* Avec french, on peut utiliser la commande \refmark.

Exemple :
le vrai appel\footnote{C'est une note en bas de
page\label{manote}} et le deuxieme appel\refmark{manote}
%%%% fin exemple %%%%

Remarque : Si vous n'utilisez pas french, cette macro \refmark
est décrite dans le cahier Gutenberg numero 15, avril
1993, page 52. Ce cahier est accessible a l'URL :
http://www.univ-rennes1.fr/pub/GUTenberg/publications

# 6.33 # Comment éviter les orphelins en début ou fin de page ?
---------------------------------------------------------------
* Il suffit d'ajouter les lignes :
%%%% debut macro %%%%
\widowpenalty=10000
\clubpenalty=10000
\raggedbottom
%%%% fin macro %%%%
dans le préambule du document.

* Pour agir localement, on peut également utiliser la commande
\enlargethispage (cf. question 6.11).

# 6.34 # Comment définir de nouveaux flottants ?
------------------------------------------------
Le package 'float' de G. Williams, disponible sur CTAN dans
/macros/latex/contrib/supported/float/, permet de définir de
nouveaux objets flottants. Il définit notamment des
environnements permettant d'encadrer des objets ou de les
séparer du reste du texte par des lignes horizontales.

Exemple :
\documentclass[11pt]{report}
\usepackage{float}
\usepackage{french}

\floatstyle{ruled} % pour que mes flottants soient séparés du
% texte par des lignes.
\newfloat{important}{htbp}{loi}[section]
% important est le nom de mon nouvel environnement
% htbp sont les options de placement de mon flottant
% loi est l'extension du fichier qui sera utilise pour
% construire la liste de mes flottants
% section est le niveau duquel dependra la numerotation
% de mes flottants
\floatname{important}{Important} % titre du caption

\begin{document}

\listof{important}{Liste des textes importants.} % titre de la
% liste de mes flottants.

\chapter{Règles de bases.}

\section{Hommes célèbres.}

Parmi les citations des hommes célèbres dans le domaine que
nous étudions actuellement, il faut retenir celle de M.
Maxime rappelée dans le cadre~\ref{imp-max}.

\begin{important}
M. Maxime a dit un jour~:
\begin{quote}
Chacun doit se faire ses propres raisons.
\end{quote}
\caption{Adage de M. Maxime.}
\end{important}

Notez que~\ldots
\end{document}
%%%% fin exemple %%%%

# 6.35 # Comment utiliser la commande \caption hors d'un
--------------------------------------------------------
environnement flottant ?
------------------------
* Pour pouvoir attacher un titre de style table à un
environnement non flottant (i.e. autre que figure ou table), il
faut définir :
%%%% debut macro %%%%
\makeatletter
\def\@captype{table}
\makeatother
%%%% fin macro %%%%

Exemple :
Texte sur les couleurs.

\begin{itemize}
\item rouge
\item vert
\item bleu
\end{itemize}
\caption{Liste des couleurs primaires.}

Texte.
%%%% fin exemple %%%%

* La macro suivante propose une autre possibilité pour obtenir
un titre de style table :
%%%% debut macro %%%%
\makeatletter
\def\captionof#1#2{{\def\@captype{#1}#2}}
\makeatother
%%%% fin macro %%%%

Exemple :
\begin{document}
un texte

\captionof{table}{\caption{un titre ici}\label{foo}}

un autre texte
\end{document}
%%%% fin exemple %%%%

* Le package 'capt-of' disponible sur CTAN dans
/macros/latex/contrib/other/misc/ définit la commande
\captionof.

# 6.36 # Comment encadrer un objet flottant ?
---------------------------------------------
* Il faut utiliser le package 'float' qui définit le style
boxed. Ce package est disponible sur CTAN dans
/macros/latex/contrib/supported/float/. Voir la question 6.34
pour un exemple plus complet.

Exemple :
Dans le préambule :
\floatstyle{ruled}
\newfloat{nom-flottant}{placement}{ext-fic}[numerotation]
dans le corps du document :
\begin{nom-flottant}

\end{nom-flottant}
%%%% fin exemple %%%%

* Si le titre accompagnant le flottant peut être à l'extérieur
du cadre désiré, on peut alors utiliser \fbox dans la
déclaration du flottant.

Exemple :
\begin{table}
\begin{center}
\fbox{
\begin{tabular}{cll}
&1 case & 2 cases \\
&3 cases & 4 cases \\\hline
Total~: & 4 cases & 6 cases \\
\end{tabular}
}
\end{center}
\caption{Titre extérieur.}
\end{table}
%%%% fin exemple %%%%

* M. Loreti propose également le package 'bigbox' ci-dessous :
%%%% debut macro %%%%
% I like to put boxes around all of my figures and algorithms in
% reports, and finally came up with my 'bigbox' environment. It
% makes a box as wide as the current text and sets the stuff
% inside with a narrower width.
%
% USAGE:
%
% \begin{figure} (or {table}, or {center}, or ...)
% \begin{bigbox}
% \begin{tabbing}
% ...
% \end{tabbing}
% \end{bigbox}
% \caption{XXX}
% \end{figure}
%
%%% BIGBOX - environment
%%%
%%% TOM SHEFFLER
%%% Carnegie Mellon, Aug 1990
%%%
%%% Make an environment for boxing a figure and setting it in
%%% a narrower width.
\newdimen\boxfigwidth % width of figure box

\def\bigbox{\begingroup
% Figure out how wide to set the box in
\boxfigwidth=\hsize
\advance\boxfigwidth by -2\fboxrule
\advance\boxfigwidth by -2\fboxsep
\setbox4=\vbox\bgroup\hsize\boxfigwidth
% Make an invisible hrule so that the box is exactly this wide
\hrule height0pt width\boxfigwidth\smallskip%
% Some environments like TABBING and other LIST environments
% use this measure of line size - \LINEWIDTH=\HSIZE-\LEFTMARGIN-\RIGHTMARGIN?
\linewidth=\boxfigwidth
}
\def\endbigbox{\smallskip\egroup\fbox{\box4}\endgroup}
%%%% fin macro %%%%

Exemple :
\documentclass[11pt]{report}
\usepackage{graphics}
\usepackage{bigbox}
\usepackage{french}
\pagestyle{empty}
\begin{document}

\begin{figure}
\begin{bigbox}
\includegraphics{../foobar.ps}
\end{bigbox}
\caption{Dessin.}
\end{figure}

\end{document}
%%%% fin exemple %%%%


# 6.37 # Comment changer l'orientation d'un caption ?
-----------------------------------------------------
La commande \rotcaption fournie avec le package 'rotating'
permet de changer l'orientation d'un caption.

Exemple :
\begin{figure}
\centering
\begin{minipage}[c]{1in}
\includegraphics[angle=90,width=\linewidth]{coco.ps}
\end{minipage}
\begin{minipage}
\rotcaption{Ma jolie figure coco}
\label{coco}
\end{minipage}
\end{figure}
%%%% fin exemple %%%%

# 6.38 # Comment mettre en page un programme ?
----------------------------------------------
Les packages 'program' (disponible sur CTAN dans
/macros/latex/contrib/supported/program/) et 'programs'
permettent de mettre en reliefs des mots clés, d'utiliser des
mathématiques dans des algorithmes, etc.

# 6.39 # Comment obtenir un espace insécable ?
----------------------------------------------
Le caractère ~ est interprété par LaTeX et permet d'obtenir un
espace insécable.

Exemple :
Comme le montre l'exemple suivant~: $a + 2 = 0$.
%%%% fin exemple %%%%

# 6.40 # Comment insérer une page blanche ?
-------------------------------------------
* Pour forcer latex à laisser une page blanche dans un document,
il faut utiliser successivement les trois commandes :
\newpage
\strut ou ~ ou \mbox{} ou \null
\newpage

Le principe est qu'il y ait quel que chose d'invisible sur la
page blanche pour que LaTeX la prenne en compte.

# 6.41 # Comment supprimer l'indentation des paragraphes ?
----------------------------------------------------------
* \noindent au début du texte permet de ne pas indenter un
paragraphe.

* Pour systématiquement supprimer l'indentation du premier
paragraphe d'une section par exemple, il faut redéfinir la
commande \section. Il faut que le 4ème paramètre de
\@startsection soit une distance négative, pour qu'il n'y ait
pas de retrait au premier paragraphe du texte qui suit :

%%%% debut macro %%%%
\makeatletter
\renewcommand\section{\@startsection {section}{1}{\z@}%
{-3.5ex \@plus -1ex \@minus -.2ex}%
%%%%%% ^^^^ (4eme parametre)
{2.3ex \@plus.2ex}%
{\normalfont\Large\bfseries}}
\makeatother
%%%% fin macro %%%%

# 6.42 # Comment modifier l'espacement entre caractères ?
---------------------------------------------------------
* Le package 'letterspace' permet de modifier l'espacement entre
mots ou caractères.

* Le package 'trackin' disponible sur CTAN dans
/macros/latex/contrib/other/tracking/ permet de jouer sur
les espacements dans les mots ou les phrases pour les ajuster
dans une longueur spécifiée.

# 6.43 # Comment supprimer certaines coupures de mots ?
-------------------------------------------------------
* Pour agir sur un mot particulier, il faut utiliser la commande
\hyphenation.

Exemple :
Visualisation des coupures :
+\showhyphens{mousehole AlGaAs GaAs GaInP AlInP}

Underfull \hbox (badness 10000) detected at line 0
[] \OT1/cmr/m/n/10 mouse-hole Al-GaAs GaAs GaInP Al-InP

Interdiction de certaines coupures :
+\hyphenation{AlGaAs GaAs GaInP AlInP}
+\showhyphens{mousehole AlGaAs GaAs GaInP AlInP}

Underfull \hbox (badness 10000) detected at line 0
[] \OT1/cmr/m/n/10 mouse-hole AlGaAs GaAs GaInP AlInP
%%%% fin exemple %%%%

* Moins propre mais tout aussi efficace, on peut inclure le mot
à ne pas couper dans une hbox.

Exemple :
Même quand il est en bout de ligne ce mot très long n'est pas
coupé CeMotTrèsLongNestPasCoupé.

Même quand il est en bout de ligne ce mot très long n'est pas
coupé \hbox{CeMotTrèsLongNestPasCoupé}.
%%%% fin exemple %%%%

* Pour empêcher LaTeX de couper les mots dans un paragraphe il
suffit de l'encadrer par les commandes \begin{sloppypar} et
\end{sloppypar}.

* Pour empêcher LaTeX de couper les mots dans tout un document,
il faut utiliser la commande \sloppy dans le préambule du
document. On peut également utiliser la déclaration :
\hyphenpenalty 10000.

* De manière globale, on peut aussi déclarer :
\DeclareFontFamily{T1}{cmr}{\hyphenchar\font=-1}

* Pour interdire la coupure de tous les mots commençant par une
majuscule, il faut utiliser : \uchyph=0 dans le préambule du
document.

* On peut également interdire la coupure des mots d'une langue
particulière dans un document multilingues en utilisant l'astuce
suivante : il suffit de créer un fichier de motifs de césures
vide, par exemple pour le russe
%%%% debut macro %%%%
%%% ruhyph.tex %%%
\patterns{}
\endinput
%%%%%%%%%%%%%%%%%%%
%%%% fin macro %%%%
et dans le fichier language.dat ajouter la ligne
russian ruhyph.tex

# 6.44 # Comment mettre en forme des équations chimiques ?
----------------------------------------------------------
* Le package 'ppchtex' disponible sur CTAN dans
/cros/context/ppchtex/ permet d'écrire des formules chimiques.

* Sur Mac ou PC, il existe MDL qui est une version freeware
d'ISIS Draw qi permet de créer ses propres structures et de les
sauver sous format eps. Pour plus de détails, consulter
http://www.mdli.com/prod/ioffer.html.

* Il existe le package 'chemsym' disponible sur CTAN dans
/macros/latex/contrib/supported/chemsym/.

* Le package 'XymTex' disponible sur CTAN dans
/macros/latex209/contrib/xymtex/ ou
/macros/latex/contrib/other/xymtex/ permet de définir des
structures chimiques.

Remarques (F. Jacquet) :
+ XyMTeX est incompatible avec le package french de mr Gaulle.
Pour utiliser les deux dans un même document, il suffit de
repasser en mode \english avant la macros puis \french après. Je
ne sais pas comment cela ca fonctionne pour les \caption mais en
théorie cela devrait marcher !
+ XyMTeX possède le gros défaut de ne pas pouvour faire de
longue chaine aliphatique si on se pas programmer le nombre de
points entre deux structures. Le plus simple dans ce cas est
donc l'emploi de Xfig (ou autre).
+ On ne peut pas imbriquer les formules ce qui gène
considérablement également son utilisation par contre pour les
cholestérol c'est très puissant !

# 6.45 # Comment mettre en page un calendrier ?
-----------------------------------------------
* Le package 'termcal' disponible sur CTAN dans
/macros/latex/contrib/supported/termcal/ permet de mettre en
page un calendrier. Il permet de préciser quels jours doivent
apparaitre et d'insérer du texte soit régulièrement à certaines
dates soit à des dates particulières.

* Le package 'calendar ' disponible sur CTAN dans
/macros/plain/contrib/ propose un ensemble de macros pour
mettre en page des calendriers.

* Un autre package 'calendar' est également disponible sur CTAN
dans /macros/latex/contrib/supported/calendar/.

* Il existe également le package 'yplan97' disponible sur CTAN
dans /macros/latex/contrib/other/yplan97/.

# 6.46 # Comment forcer un caption sur plusieurs lignes ?
---------------------------------------------------------
Pour forcer un retour à la ligne dans un caption, il faut
utiliser \caption[text1]{text2 \\\hspace{\linewidth} text3}.

Remarque : text1 est le texte qui apparaîtra dans la liste des
figures ou des tableaux.

# 6.47 # Comment générer des barres de modification dans un
-----------------------------------------------------------
document ?
----------
Il suffit d'utiliser le package 'changebar' disponible sur CTAN
dans /macros/latex/contrib/supported/changebar/.

Exemple :
dans le préambule :
\usepackage[outerbars]{changebar} % permet de positionner les
% barres en marge externe
\setcounter{changebargrey}{20} % permet de fixer le niveau de
% gris des barres

et dans le document :
\begin{changebar}
texte..
\end{changebar}

ou

ancien texte ancien texte ancien texte \cbstart nouveau
texte nouveau texte nouveau texte \cbend ancien texte ancien
texte ancien texte ancien texte
%%%% fin exemple %%%%

# 6.48 # Comment encadrer du texte verbatim ?
---------------------------------------------
* Le package 'moreverb' propose un environnement encadré.

Exemple :
\begin{boxedverbatim}
La commande \LaTeX permet d'appeler le logo
LaTeX.
\end{boxedverbatim}
%%%% fin exemple %%%%

# 6.49 # Comment écrire du texte en forme de losange ou autre ?
---------------------------------------------------------------
Le package 'shapepar' disponible par ftp à ftp.loria.fr dans
/pub/ctan/macros/latex/contrib/other/misc/ définit des
environnements losange, coeur, etc.

Exemple :
\diamondpar{mon paragraphe en forme de losange.}
%%%% fin exemple %%%%

# 6.50 # Comment isoler les flottants par un trait horizontal ?
---------------------------------------------------------------
* Pour que les flottants qui sont renvoyés en début
(respectivement en fin) de page soient isolés du texte par un
trait horizontal en dessous (respectivement au dessus), il faut
activer les options suivantes dans le préambule du document :
\let\topfigrule\hrule
\let\botfigrule\hrule

* Si leur comportement n'est pas satisfaisant, on peut le
redéfinir :
\newcommand{\topfigrule}{%
\vspace*{3pt}\noindent\rule{\linewidth}{0.4pt}\vspace{-3.4pt}}
\newcommand{\botfigrule}{%
\vspace*{3pt}\noindent\rule{\linewidth}{0.4pt}\vspace{-3.4pt}}

# 6.51 # Comment insérer un espace vertical dans une page ?
-----------------------------------------------------------
Il suffit d'utiliser la commande \vspace et de préciser en
argument la hauteur de l'espace voulu.

Exemple :
Texte avant.

\vspace{2cm}

Texte après.
%%%% fin exemple %%%%

Remarque : la commande \vspace* force l'insertion d'un espace
vertical même si ce dernier se situe sur un
changement de page.

# 6.52 # Comment insérer un espace horizontal dans un texte ?
-------------------------------------------------------------
De la même manière que pour les espaces verticaux, il existe la
commande \hspace pour insérer un espace horizontal dans un
texte.

Exemple :
blabla bla \hspace{3cm} bli bli bli
%%%% fin exemple %%%%

Remarque : la commande \hspace* force l'insertion d'un espace
même en début ou en fin de ligne.

# 6.53 # Comment définir un style de paragraphe ?
-------------------------------------------------
* Pour agir de manière globale (sur tout le document), les
paramètres de définition d'un paragraphe sont :
+ \parindent pour fixer la longueur des indentations
+ \parskip pour gérer l'espace entre les paragraphes.

Exemple :
\setlength{\parindent}{1cm}
\setlength{\parskip}{1ex plus 0.5ex minus 0.2ex}
%%%% fin exemple %%%%

* Pour forcer (supprimer) localement l'indentation d'un
paragraphe, il existe la commande \(no)indent.

# 6.54 # À quoi sert la commande \stretch ?
-------------------------------------------
La commande \stretch{x} permet d'insérer un espace élastique qui
va s'ajuster de manière à forcer l'occupation de toute une
ligne (en mode horizontal) ou de toute une page (en mode
vertical). Le paramètre x est appelé facteur d'élasticité, il
intervient dès que plusieurs commandes \stretch sont appelées
sur la même ligne.

Exemples :
+ bla\hspace{\stretch{1}} bla bla \hspace{\stretch{2}} bla bla
bla

+ un texte normal \vspace{\stretch{1}} et un texte qui finit de
remplir la ligne courante et qui se poursuit tout en bas de la
page sur la dernière ligne.
%%%% fin exemple %%%%

# 6.55 # Comment justifier un paragraphe à gauche ou à droite ?
---------------------------------------------------------------
* Les environnements flushleft et flushright permettent de
justifier à gauche ou à droite une portion de texte.

Exemple :
\begin{flushleft}
Texte aligné à gauche. Texte aligné à gauche. Texte aligné à
gauche. Texte aligné à gauche. Texte aligné à gauche. Texte
aligné à gauche. Texte aligné à gauche. Texte aligné à
gauche. Texte aligné à gauche. Texte aligné à gauche. Texte
aligné à gauche.
\end{flushleft}
%%%% fin exemple %%%%

* Il existe également les commandes \raggedright et \raggedleft.

Exemple :
{\raggedleft
Colle à droite. Colle à droite. Colle à droite. Colle à droite.
Colle à droite. Colle à droite. Colle à droite.

Colle à droite. Colle à droite. Colle à droite. Colle à droite. Colle à
droite. Colle à droite. }
%%%% fin exemple %%%%

* Le package 'ragged2e' disponible sur CTAN dans
/macros/latex/contrib/supported/ragged2e offre en plus des
commandes (\Centering, \RaggedLeft, et \RaggedRight) et des
environnements qui permettent de mieux gérer les césures.

# 6.56 # Comment mettre en page des citations ?
-----------------------------------------------
Il existe deux environnements de gestion des citations :
+ quote pour les citations courtes,
+ quotation pour les citations longues.

Exemple :
\begin{quote}
L'environnement \texttt{quote} n'indente pas ces paragraphes
par contre l'espace vertical entre ces derniers est supérieur
à celui d'un texte standard.

La preuve~!
\end{quote}

\begin{quotation}
L'environnement \texttt{quotation} indente la première ligne
de ses paragraphes et sépare ses paragraphe d'un espace
vertical standard.

La preuve~!

\end{quotation}

Dans les deux cas les marges droite et gauche sont plus
importante que celles d'un texte standard.
%%%% fin exemple %%%%

# 6.57 # Comment insérer un texte sans que LaTeX le mette en
------------------------------------------------------------
forme ?
-------
L'environnement verbatim permet d'insérer un texte tel quel sans
que LaTeX ne le traite. Il permet notamment de faire apparaître
des commandes LaTeX.

Exemple :
\begin{verbatim}
La commande \LaTeX permet d'imprimer le logo LaTeX.
\end{verbatim}
%%%% fin exemple %%%%

* La commande \verb?texte? permet de faire la même chose
localement. Le premier caractère ? permet de marquer le début du
mode verbatim et le second en marque la fin. Il peut être
remplacé par n'importe quel autre caractère sauf un espace ou *.

# 6.58 # Comment visualiser des espaces ?
-----------------------------------------
* L'environnement verbatim* et la commande \verb* permettent de
visualiser les espaces insérés dans un texte en les remplacant
par un caractère spécial en forme de u.

Exemple :
Je mange \verb*? ? pomme.
%%%% fin exemple %%%%

* LaTeX propose la commande \textvisiblespace.

* En TeX, c'est le caractère 32 de la fonte cmtt, qui est
défini dans le source du TeXBook par :
\def\]{\leavevmode\hbox{\tt\char`\ }} % visible space

Vous pouvez donc définir:
\def\vs{\leavevmode\hbox{\tt\char`\ }} % visible space
et \vs donnera ce fameux caractère.

# 6.59 # Comment insérer du code LaTeX dans un document LaTeX ?
---------------------------------------------------------------
* Le package 'oz' offre un environnement example qui permet en
ne tapant qu'une seule fois le code d'avoir côte à côte le code
LaTeX et son résultat après compilation.

Exemple :
\begin{example}
La commande \LaTeX permet d'appeler le logo LaTeX.
\end{example}
%%%% fin exemple %%%%

* La solution la plus classique consiste à faire :
\hbox{\vtop{\hsize 0.5\hsize le permier texte}\vtop{\hsize
0.5\hsize{\begin{verbatim} le même texte \end{verbatim}}}

# 6.60 # Comment écrire du texte en couleurs ?
----------------------------------------------
* Voir à ce sujet le Cahier GUTenberg numéro 16 (février 1994,
entièrement consacré à ce problème) et l'article de M. Goossens
et M. Jouhet dans Cahier GUTenberg 21 (juin 1995, pages 30-52),
accessible à l'URL :
http://www.univ-rennes1.fr/pub/GUTenberg/publications

* Le package 'color' est disponible sur CTAN dans
/macros/latex/packages/graphics/. Il permet de coloriser le
texte ou le fond du document.

Exemple :
\textcolor{red}{Important}
%%%% fin exemple %%%%

* pstricks disponible sur CTAN dans /graphics/pstricks/ est un
ensemble de macros PostScript compatibles avec Plain
TeX, LaTeX, AmSTeX, and AmS-LaTeX. Il comprend notamment
des macros pour la colorisation, la gestion des graphiques, le
dessin de camembert, d'arbres, etc.

* Voir également la question 6.25 : Comment griser le fond
(background) d'un paragraphe ?

# 6.61 # Comment construire un arbre ?
--------------------------------------
* Le package 'qobitree', est disponible sur CTAN dans
/macros/latex/contrib/other/qobitree/.

* Le package 'treesvr' est également disponible sur CTAN dans
/macros/latex/contrib/supported/treesvr/.

Exemple :
\begin{picture}(100,100)
\setlength\unitlength{2mm}
\branchlabels ABC % 012 is the default
\root(2,10) 0. % root at absolute coordinate (2,10)
% its (internally used) label is 0
% the space before the 0 obligatory
\branch2{16} 0:1,2. % node 0 has children 1 and 2
% the text "16" is written above it
% space optional, :,. obligatory
\leaf{4}{$u_1$} 1. % node 1 is a leaf
% "4" written above, "$u_1$" at right
\branch2{12} 2:3,7. % branch to node 3 goes up labeled A
\tbranch2{9} 3:4,5,6.
\leaf{4}{$u_3$}4.
\leaf{3}{$u_4$}5.
\leaf{2}{$u_5$}6.
\leaf{3}{$u_2$} 7.
\end{picture}
%%%% fin exemple %%%%

* Le package 'treetex' disponible sur CTAN dans
/macros/latex209/contrib/trees/tree_tex/ permet de metttre en
page des arbres sous LaTeX.

* Les packages 'epic', 'eepic', 'ecltree' permettent de créer des
arbres (les trois packages sont nécessaires pour une
utilisation avec Latex2e pour garder la compatibilité avec
Latex2.09).

Exemple :
\documentstyle[epic,eepic,ecltree]{article}
\begin{document}

\begin{bundle}{racine}
\chunk{feuille 1}
\chunk{feuille 2}
\drawwith{\dottedline{3}}
\chunk{feuille 3}
\drawwith{\drawline}
\chunk{feuille 4}
\end{bundle}

\end{document}
%%%% fin exemple %%%%

# 6.62 # Comment mettre en page un poème ?
------------------------------------------
* Il existe l'environnement verse. Il gère les marges, les
retours à la ligne dans une strophe se font par \\ et les
strophes sont séparés par des lignes blanches.

Exemple:
\begin{verse}
J'ai un poney gris, \\
Qui galope à travers la prairie.

Grignote, grignote dans ma main, \\
La carotte rousse du jardin.

Mes cousins ont un poney blanc, \\
Qui parcourt chemins et champs.
\end{verse}
%%%% fin exemple %%%%

* Ci-dessous un exemple de style de JHB Nijhof
(nijh...@aston.ac.uk) :

Exemple :
\documentstyle[a4,12pt]{article}

\def\testline{\par\noindent\hbox to 0pt{\hss*\hss}\hfill
\hbox to 0pt{\hss*\hss}\hfill\hbox to 0pt{\hss*\hss}\par}
\def\centerstar{\par\medskip\noindent\hbox to\hsize%
{\hss*\hss}\par\medskip}

\newbox\poembox
\newbox\widebox
\newdimen\centerx
\newcount\linecount
\newdimen\poemleftmargin
\def\newpoem{\setbox0=\box\poembox \setbox0=\box\widebox
\linecount=0} % empty boxes
\newpoem % probably not necessary
\def\poemline#1{\setbox0=\hbox{\strut #1}%
\setbox\poembox=\vbox{\unvbox\poembox\copy0}%
\setbox\widebox=\hbox{\unhbox\widebox\copy0}%
\advance \linecount 1}
\def\setpoem{% you'll want a clearpage here
\centerx=\wd\widebox \divide\centerx\linecount \divide\centerx 2%
%now centerx is the x of the center of gravity
\poemleftmargin=0.5\hsize \advance \poemleftmargin-\centerx
\noindent\kern\poemleftmargin\box\poembox
\par
\newpoem}

\begin{document}

\testline % for viewing the margins/ middle
\poemline{Rozen verwelken}
\poemline{schepen vergaan}
\poemline{maar onze liefde}
\poemline{zal blijven bestaan}
\setpoem

\centerstar

\poemline{Rozen verwelken}
\poemline{schepen vergaan}
\poemline{maar onze liefde zal blijven bestaan}
\setpoem

\centerstar

\poemline{Rozen verwelken, schepen vergaan}
\poemline{maar onze liefde zal blijven bestaan}
\setpoem

\centerstar

\poemline{Rozen verwelken}
\poemline{\qquad schepen vergaan}
\poemline{maar onze liefde}
\poemline{\qquad zal blijven bestaan}
\setpoem

\centerstar

\end{document}
%%%% fin exemple %%%%

# 6.63 # Comment aligner des paragraphes ?
------------------------------------------
On peut utiliser la commande \parindent.

Exemple :
{\settowidth{\parindent}
{Notes~:\ }

\makebox[0pt][r]
{Notes~:\ }La première note pour dire que ...

La seconde pour préciser que ...
}
%%%% fin exemple %%%%

# 6.64 # Comment construire une liste d'éléments ?
--------------------------------------------------
Il existe trois environnements de liste par défaut :
+ l'environnement itemize
+ l'environnement description
+ l'environnement enumerate

Exemples :
\begin{itemize}
\item un élément
\item un autre élément
\end{itemize}

\begin{description}
\item[Genre] Le genre peut être féminin ou masculin
\item[Nombre] Le nombre peut être singulier ou pluriel
\end{description}

\begin{enumerate}
\item premier élément
\item deuxième élément
\end{enumerate}
%%%% fin exemple %%%%

# 6.65 # Comment faire une note dans une marge ?
------------------------------------------------
On peut utiliser la commande \marginpar{note marge}.

Exemple :
La valeur du paramètre temps est fixée à
12~minutes\marginpar{AC}. En tenant compte de cette hypothèse
les résultats obtenus sont les suivants~:

~\ldots
%%%% fin exemple %%%%

Remarque : Pour inverser les notes dans les marges
(droite/gauche) il suffit de mettre dans le
préambule : \reversemarginpar.

# 6.66 # Comment faire une page de garde ?
------------------------------------------
Les commandes permettant de définir une page de garde sont :
+ \title pour un titre
+ \author pour le ou les auteurs (dans ce dernier cas leurs noms
sont séparés par la commande \and)
+ \date pour la date
+ \thanks utilisée dans l'une des commandes ci-dessus permet de
faire référence à une note de bas de page.

La page de garde est ensuite générée par la commande \maketitle
appelée dans le corps du document (généralement juste après la
balise \begin{document}).

Exemple :
\documentclass{report}
\usepackage{french}

\newlength{\larg}
\setlength{\larg}{14.5cm}

\title{
{\rule{\larg}{1mm}}\vspace{7mm}
\begin{tabular}{p{4cm} r}
& {\Huge {\bf {FAQ} \LaTeX{} française}} \\
& \\
& {\huge Pour débutants et confirmés}
\end{tabular}\\
\vspace{2mm}
{\rule{\larg}{1mm}}
\vspace{2mm} \\
\begin{tabular}{p{11cm} r}
& {\large \bf Version 2.0} \\
& {\large \today}
\end{tabular}\\
\vspace{5.5cm}
}
\author{\begin{tabular}{p{13.7cm}}
Marie-Paule Kluth
\end{tabular}\\
\hline }
\date{}

\begin{document}
\maketitle

Voici mon document.

\end{document}
%%%% fin exemple %%%%

# 6.67 # Comment utiliser \thanks dans chaque titre de chapitre ?
-----------------------------------------------------------------
Pour associer des remerciements dans chaque chapitre d'un thèse,
par exemple, il faut définir sa propre macro à base de
\footnote.

Exemple (R. Fairbairns) :
\documentclass{report}
\usepackage{french}

\newcommand\thankschapter[2]{%
% arg 1 is chapter title
% arg 2 is `thanks' text
\edef\savefootnote{\thefootnote}
\renewcommand\thefootnote{\fnsymbol{footnote}}
\chapter[#1]{#1\footnote[1]{#2}}
\renewcommand\thefootnote{\arabic{footnote}}
\setcounter{footnote}{\savefootnote}
}
\begin{document}
\chapter{Préface.}{\textit{Un grand merci à M. Nigaudon pour
avoir accepter de préfacer ce livre.\\}}

Ce livre traitant de \LaTeX,~\ldots

\end{document}
%%%% fin exemple %%%%

# 6.68 # Comment mettre un résumé et un abstract dans un
--------------------------------------------------------
document ?
----------
* Le package 'french' propose les environnements resume et
abstract.

Exemple (textes sur la même page) :
\documentclass{article}
\usepackage{french}
\pagestyle{empty}
\begin{document}
\begin{resume} résumé en français \end{resume}
\english
\begin{abstract} the same in english \end{abstract}
\french

Cet article~\ldots
\end{document}
%%%% fin exemple %%%%

Remarque : \resume n'est qu'un alias de \abstract.

Exemple (chaque résumé est sur une page séparée) :
\begin{english}
\begin{abstract}
english text
\end{abstract}
\end{english}

\begin{french}
\begin{abstract}
Texte français
\end{abstract}
\end{french}
%%%% fin exemple %%%%

* Le package 'babel' peut également être utilisé.

Exemple :
\usepackage[francais,english]{babel}
...
\selectlanguage{francais}
resume : je parle français
....
\selectlanguage{english}
abstract : I speak english
....
%%%% fin exemple %%%%

* Autre solution qui ne permet toutefois pas d'avoir les deux
textes sur la même page :

%%%% debut macro %%%%
\renewcommand{\abstractname}{Résumé}
\begin{abstract} résumé en français \end{abstract}
\renewcommand{\abstractname}{Abstract}
\begin{abstract} the same in english \end{abstract}
%%%% fin macro %%%%

# 6.69 # Comment définir de nouvelles couleurs ?
------------------------------------------------
Il existe deux repères pour cela :
+ cyan, magent, yellow, black et la commande :
\definecolor{nouveau_nom}{cmyk}{w,x,y,z}
\newcmykcolor{le_nom_de_la_couleur}{w x y z}
avec w,x,y,z entre 0.0 et 1.0

+ red,green, blue et la commande :
\definecolor{nouveau_nom}{rgb}{a,b,c}
ou : \newrgbcolor{le_nom_de_la_couleur}{a b c}
avec a,b,c entre 0.0 et 1.0

+ il existe également :
\newhsbcolor{le_nom_de_la_couleur}{num1 num2 num3}
qui permet de jouer sur le << hue >>, la saturation et la
luminosite.

# 6.70 # Comment placer des figures face à face en recto-verso ?
----------------------------------------------------------------
Dans un document recto-verso, pour s'assurer qu'une figure
apparaisse sur la page de gauche et une autre sur la page de
droite en face, il faut utiliser le package 'dpfloat' disponible
à : http://weber.u.washington.edu/~fox/tex/dpfloat.html.

# 6.71 # Quelles sont les différences entre a4paper et
------------------------------------------------------
letterpaper ?
-------------
L'option a4paper exécute :
\setlength\paperheight {297mm}
\setlength\paperwidth {210mm}

L'option par défaut est letterpaper qui fait
\setlength\paperheight {11in}
\setlength\paperwidth {8.5in}

Remarque (J. Kanze) : Il faut remarquer que la différence n'est
pas énorme : les défauts en mm sont 279x216. En fait,
sur le petit test que j'ai fait, les coupures de
lignes se trouvaient exactement aux mêmes endroits --
en revanche, chaque page contenait bien quelques
lignes de plus avec a4paper.

# 6.72 # Comment couper une chaîne de caractères ?
--------------------------------------------------
* Pour forcer la coupure d'une chaîne de caractères, il faut
utiliser la commande \- à l'endroit où la chaîne doit être
coupée.

Exemple :
Je décide que le dernier mot de cette ligne soit coupé en deux
ici:~ do\-cumentation. N'est ce pas beau~?
%%%% fin exemple %%%%

* Pour indiquer à LateX comment couper certain mots, on peut
utiliser la commande \hyphenation.

Exemple :
\hyphenation{do-cu-ment}
Je décide que le dernier mot de cette ligne soit coupé en deux
ici:~ documentation. N'est ce pas beau~?
%%%% fin exemple %%%%

# 6.73 # Comment modifier l'espace entre une figure et sa
----------------------------------------------------------
légende ?
--------
Les deux variables qui définissent respectivement l'espace avant
une légende et l'espace après sont : \abovecaptionskip et
\belowcaptionskip.

Exemple de redimensionnement :
\setlength\abovecaptionskip{0.25ex}
%%%% fin exemple %%%%


================================================================
[7] GESTION DES TABLEAUX
================================================================

# 7.1 # Comment faire un tableau ?
----------------------------------
En mode texte, il faut utiliser l'environnement tabular. Chaque
colonne est ensuite décrite par r, l ou c pour obtenir une
colonne de texte justifié à droite, à gauche ou centré. Chaque
déclaration de colonne peut être délimitée par une séparation
verticale : | ou rien. Une ligne horizontale entre deux lignes
s'obtient par la commande \hline. Le contenu de chaque colonne
est séparé de celui de la suivante par le symbole &. Le
changement de ligne est obtenu par la commande \\.

Lorsque plusieurs colonnes adjacentes ont la même définition, on
peut grouper leurs déclarations avec la commande
*{nb_colonnes}{déclaration}

Exemple :
\begin{tabular}{|*{2}{c|}l r|}
\hline
une & deux & trois & quatre \\
case centrée & encore centrée & à gauche & à droite \\
\hline
\end{tabular}
%%%% fin exemple %%%%

# 7.2 # Comment gérer les tableaux de plus d'une page ?
-------------------------------------------------------
* Le package 'supertabular' permet de gérer automatiquement (ou
presque) les coupures de tableaux. Il est disponible sur CTAN
dans /macros/latex/contrib/supported/supertabular/.
Il calcule la longueur du tableau à chaque \\ et vérifie si la
fin de page est atteinte. Si oui, il coupe le tableau. Les
commandes à utiliser sont :
+ \tablefirsthead{...} pour définir le contenu de la première
occurence de la tête du tableau. Cette commande est optionnelle.
+ \tablehead{...} pour définir le contenu de la tête des
tableaux suivants.
+ \tabletail{...} définit le contenu de la ligne qui doit être
ajoutée avant une coupure
+ \tablelasttail{...} définit le contenu de la dernière ligne
du tableau. Cette commande est optionnelle.
+ \topcaption{...}, \bottomcaption{...} et \tablecaption{...}
permettent de nommer un tableau soit au début ou à la fin du
tableau. \tablecaption{...} est l'option par défaut.

Exemple :
\begin{center}
\tablefirsthead{\hline \multicolumn{1}{|c}{Nombre}
& \multicolumn{1}{c}{Nombre$^2$}
& Nombre$^4$
& \multicolumn{1}{c|}{Nombre!} \\ \hline}
\tablehead{\hline \multicolumn{4}{|l|}{\small\sl continued
from previous page}\\
\hline \multicolumn{1}{|c}{ Nombre}
& \multicolumn{1}{c}{Nombre$^2$}
& Nombre$^4$
& \multicolumn{1}{c|}{Nombre!} \\ \hline}
\tabletail{\hline\multicolumn{4}{|r|}{\small\sl Suite page
suivante~\ldots}\\\hline} \tablelasttail{\hline}
\bottomcaption{Fin.}
\par
\begin{supertabular}{| r@{\hspace{6.5mm}}|
r@{\hspace{5.5mm}}| r | r|}
1 & 1 & 1 & 1 \\
2 & 4 & 16 & 2 \\
3 & 9 & 81 & 6 \\
4 & 16 & 256 & 24 \\
5 & 25 & 625 & 120 \\
6 & 36 & 1296 & 720 \\
7 & 49 & 2401 & 5040 \\
8 & 64 & 4096 & 40320 \\
9 & 81 & 6561 & 362880 \\
10 & 100 & 10000 & 3628800 \\
11 & 121 & 14641 & 39916800 \\
12 & 144 & 20736 & 479001600 \\
13 & 169 & 28561 & 6.22702080E+9 \\
14 & 196 & 38416 & 8.71782912E+10\\
15 & 225 & 50625 & 1.30767437E+12\\
16 & 256 & 65536 & 2.09227899E+13\\
17 & 289 & 83521 & 3.55687428E+14\\
18 & 324 & 104976 & 6.40237370E+15\\
19 & 361 & 130321 & 1.21645100E+17\\
20 & 400 & 160000 & 2.43290200E+18\\
\hline
1 & 1 & 1 & 1 \\
2 & 4 & 16 & 2 \\
3 & 9 & 81 & 6 \\
4 & 16 & 256 & 24 \\
5 & 25 & 625 & 120 \\
6 & 36 & 1296 & 720 \\
7 & 49 & 2401 & 5040 \\
8 & 64 & 4096 & 40320 \\
9 & 81 & 6561 & 362880 \\
10 & 100 & 10000 & 3628800 \\
11 & 121 & 14641 & 39916800 \\
12 & 144 & 20736 & 479001600 \\
13 & 169 & 28561 & 6.22702080E+9 \\
14 & 196 & 38416 & 8.71782912E+10\\
15 & 225 & 50625 & 1.30767437E+12\\
16 & 256 & 65536 & 2.09227899E+13\\
17 & 289 & 83521 & 3.55687428E+14\\
18 & 324 & 104976 & 6.40237370E+15\\
19 & 361 & 130321 & 1.21645100E+17\\
20 & 400 & 160000 & 2.43290200E+18\\
\hline
1 & 1 & 1 & 1 \\
2 & 4 & 16 & 2 \\
3 & 9 & 81 & 6 \\
4 & 16 & 256 & 24 \\
5 & 25 & 625 & 120 \\
\end{supertabular}
\end{center}
%%%% fin exemple %%%%

* Le package 'longtable', de D. Carlisle, disponible sur
CTAN dans macros/latex/packages/tools/, fonctionne de la même
manière mais mieux et permet de définir une taille de tableau
commune sur toutes les pages. Les commandes associées à ce
package sont \endfirsthead, \endhead, \endfoot et \endlastfoot.

\begin{longtable}
{|p{0.2\linewidth}|p{0.2\linewidth}|p{0.2\linewidth}|}
\hline
Premiere colonne & Deuxieme & Troisieme \endfirsthead
\hline
Premiere & Deuxieme & Troisieme \\
\multicolumn{3}{|p{0.6666\linewidth}|}{Suite ... } \\
\endhead
\hline
\multicolumn{3}{|p{0.6666\linewidth}|}{Suite page suivante}
\\ \hline \endfoot \hline
\multicolumn{3}{|p{0.6666\linewidth}|}{C'est fini} \\
\hline
\endlastfoot \hline
1 & 1 & 1 \\
2 & 4 & 16 \\
3 & 9 & 81 \\
1 & 1 & 1 \\
2 & 4 & 16 \\
3 & 9 & 81 \\
1 & 1 & 1 \\
2 & 4 & 16 \\
3 & 9 & 81 \\
1 & 1 & 1 \\
2 & 4 & 16 \\
3 & 9 & 81 \\
\end{longtable}

au lieu de faire des calculs de largeurs de colonne, on peut
aussi mettre un \setlongtables dans le préambule et déclarer
ses tableaux comme à l'habitude en \begin{longtable}{|c|c|c|}
et LaTeX se charge du reste. Dans ce cas, l'ajustement des
colonnes peut nécessiter plusieurs (jusqu'à trois) compilations
enchaînées.

* Le package 'ltxtable' de D. Carlisle, disponible sur CTAN
dans /macros/latex/contrib/supported/carlisle/ permet de
profiter des fonctionnalités de tabularx et de longtable.

# 7.3 # Comment modifier l'orientation d'un tableau ?
-----------------------------------------------------
* On peut utiliser la commande \rotatebox du package 'graphics'
de D. Carlisle. Ce package est disponible sur CTAN dans
/macros/latex/packages/graphics/.

Exemple :
\rotatebox{90}{
\begin{tabular}{|c|c|}
\hline
salut & coucou \\
\hline
\end{tabular}
}
%%%% fin exemple %%%%

* Le style 'lscape' de D. Carlisle marche aussi avec
l'environnement longtable (du même).

* De même l'environnement sidewaystable du package 'rotating'
permet d'inclure des tableaux en mode landscape. Le sens de
rotation peut être changé par l'option counterclockwise.

Exemple :
\documentclass{report}
\usepackage[counterclockwise]{rotating}
\usepackage{french}
\pagestyle{empty}
\begin{document}
\rotatebox{90}{
\begin{tabular}{|c|c|}
\hline
vu & à voir \\
de Latour & Van Gogh \\
\hline
\end{tabular}
}
\end{document}
%%%% fin exemple %%%%

* Le package 'rotfloat' disponible sur CTAN dans
/macros/latex/contrib/supported/rotfloat/ permet de gérer la
rotation des flottants.

# 7.4 # Comment nommer un tableau ?
-----------------------------------
Pour pouvoir associer une légende (\caption) à un tableau, il
suffit de l'encapsuler dans un environnement table.

Exemple :
\begin{table}[htbp]
\begin{center}
\begin{tabular}{|c|c|}
\hline
un & deux \\
\hline
trois & quatre \\
\hline
\end{tabular}
\caption{Nom du tableau. \label{table-nom}}
\end{center}
\end{table}
%%%% fin exemple %%%%

# 7.5 # Comment modifier l'épaisseur des lignes d'un tableau ?
--------------------------------------------------------------
* Pour obtenir des lignes horizontales fines ou épaisses dans un
tableau, il faut utiliser les commandes
\setlength{\doublerulesep}{\arrayrulewidth} dans l'entête du
document puis dans le tableau
\\\hline pour une ligne fine
\\\hline\hline pour une ligne épaisse

* Autre solution : la macro de A. Kessi (alain...@psi.ch)

%%%% debut macro %%%%
\makeatletter
\def\hlinewd#1{%
\noalign{\ifnum0=`}\fi\hrule \@height #1 %
\futurelet\reserved@a\@xhline}
\makeatother
%%%% fin macro %%%%

Exemple :
\begin{tabular}{|l|r|} \hline
premier & 1 \\ \hline
second & 2 \\ \hlinewd{5pt}
total & 3 \\ \hline
\end{tabular}
%%%% fin exemple %%%%

Même chose pour les lignes verticales.
%%%% debut macro %%%%
\begin{tabular}{@{\,\vrule width 5pt\,}c|c|}
%%%% fin macro %%%%

* Le package 'easytable' disponible sur CTAN dans
/macros/latex/contrib/supported/easy/ propose différents styles
de lignes de séparation des cellules d'un tableau.

# 7.6 # Comment griser des cellules d'un tableau ?
--------------------------------------------------
* Le package 'colortab' disponible par ftp à princeton.edu dans
/pub/tvz/ ou sur CTAN dans /graphics/pstricks/generic/
(accompagné de sa doc dans /graphics/pstricks/origdoc/) permet
de griser ou de mettre en couleur certaines parties de tableau.

Remarque : ColorTab n'est pas actuellement maintenu par Van
Zandt et l'extension LongTable, avec laquelle il
était compatible, ne fonctionne plus.

* Il est possible également d'utiliser le package 'shade'
disponible sur CTAN dans /macros/generic/.

* D. Carlisle propose également le package 'colortbl' sur CTAN
dans /macros/contrib/supported/carlisle/. Ce package permet
non seulement de gérer de la couleur mais il est en outre
compatible avec le package 'longtable'. Il nécessite les
packages 'array' et 'color'.

Exemple :
\begin{tabular}{|>{\columncolor[gray]{.8}}l%
|>{\color{white}\columncolor[gray]{.2}}r|}
\hline
UN & DEUX \\
TROIS & QUATRE \\
\hline
\end{tabular}
%%%% fin exemple %%%%

# 7.7 # Comment changer la fonte d'une colonne ?
------------------------------------------------
Pour préciser une fonte spécifique dans une colonne d'un
tableau, on peut utiliser le package 'array'. Ce dernier est
disponible sur CTAN dans /macros/latex/packages/tools/.

Exemple :
\begin{tabular}{>{\bfseries}l >{\slshape}r c}
gras & penché & normal \\
\end{tabular}
%%%% fin exemple %%%%

# 7.8 # Comment créer des notes de bas de page dans un tableau ?
----------------------------------------------------------------
* Il suffit d'encapsuler le tableau dans un environnement
minipage et pour supprimer le trait de séparation des notes de
bas de page de déclarer : \renewcommand{\footnoterule}{}.

Exemple :
\begin{minipage}[t]{5cm}
\renewcommand{\footnoterule}{} % permet de supprimer le
% trait de séparation
\begin{tabular}{|c|c|}
\hline
Air & Terre\footnote{Feu}\\
\hline
\end{tabular}
\end{minipage}
%%%% fin exemple %%%%

Remarque : dans ce cas, la note apparaît juste après le tableau,
dans l'environnement minipage.

* On peut également utiliser :
\footnote[cptr]{texte.}
toujours dans un environnement minipage, et gérer soit même ses
compteurs (cptr est un entier alors que les marques qui
apparaîssent dans le tableau sont les lettres de l'alphabet).

Remarque : ici aussi la note apparaît en dessous du tableau.

* Il existe également deux commandes spécifiques \footnotemark[]
et \footnotetext[]{}. \footnotemark permet de gérer le compteur
de notes et \footnotetext permet d'insérer le texte
correspondant en bas de page.

Exemple :
\begin{tabular}{|c|c|}
\hline
donnee1\footnotemark[1] & donnee2\footnotemark[2] \\
\hline
\end{tabular}
\footnotetext[1]{Note associ\'ee \`a la donn\'ee 1.}
\footnotetext[2]{Note associ\'ee \`a la donn\'ee 2.}
%%%% fin exemple %%%%

Remarque : ici les notes apparaîssent bien dans le bas de page.

# 7.9 # Comment écrire un texte sur plusieurs colonnes ?
--------------------------------------------------------
Il faut utiliser la commande :
\multicolumn{nb_colonnes}{alignement}{Texte}

Exemple :
\begin{tabular}[b]{|l|c|}
\hline
\multicolumn{2}{|c|}{Texte sur 2 colonnes} \\
\hline \hline
donnée1 & donnée2 \\
A & B \\
\hline
\end{tabular}
%%%% fin exemple %%%%

Remarque : dans le cas où le nombre de colonnes à couvrir est
égal à 1, cette commande peut être utilisée pour
modifier l'alignement (c, l, r, |, etc) d'une
cellule.

# 7.10 # Comment passer certaines cellules en reverse vidéo ?
------------------------------------------------------------
Le package 'color' disponible sur CTAN dans
/macros/latex/packages/graphics/ permet entre autres de le
faire. En fait, il permet de définir des couleurs de texte et de
fond de page.

Exemples :
\tabcolsep=30pt
\begin{tabular}{|c|c|c|}
\hline
1&&\\
&\colorbox{black}{\strut{\color{white}Coucou}}&\\
&&2\\
\hline
\end{tabular}

ou (LaTeX2e)

\newlength\Coucoulength
\settowidth\Coucoulength{Coucou Coucou Coucou}

\begin{tabular}{|c|c|c|}
\hline
1&Coucou Coucou Coucou &\\
&\colorbox{black}{\makebox[\Coucoulength][c]{\color{white}Coucou}}
&\\
&&2\\
\hline
\end{tabular}
%%%% fin exemple %%%%

# 7.11 # Comment fixer la largeur d'une colonne ?
-------------------------------------------------
* p{largeur} dans les descripteurs de colonne permet de fixer la
largeur d'une colonne. Dans ce cas par défaut le texte est
justifié à gauche. Les commandes \centering et \raggedright
permettent respectivement de le centrer ou de le justifier à
droite.

Exemple :
\begin{center}
\begin{tabular}{|c|l|p{4cm}|r|}
\hline
centré & à gauche & largeur fixe & à droite \\
bla bla bla & bla bla bla & bla bla bla & bla bla bla \\
\hline
\end{tabular}
\end{center}
%%%% fin exemple %%%%

* Avec le package 'array', disponible sur CTAN dans
/macros/latex/packages/tools/, il faut utiliser l'option
m{largeur}.

* Le package 'easytable' disponible sur CTAN dans
/macros/latex/contrib/supported/easy/ permet facilement d'écrire
des tableaux dont les colonnes ont une largeur fixe.

# 7.12 # Comment écrire un texte sur plusieurs lignes ?
-------------------------------------------------------
Il existe le package 'multirow' disponible sur CTAN dans
/macros/latex209/contrib/misc/. Il permet d'écrire un texte à
cheval sur deux lignes.

Exemple :
\begin{table}[htbp]
\begin{center}
\begin{tabular}{|c|c||c|c|c|}
\hline
\multirow{2}{0.5cm}{k} & \multirow{2}{0.5cm}{$p_G$} &
\multicolumn{2}{c|}{test} \\
\cline{3-4}
& & DADWRD & RARWRD \\
\hline
2 & $1$ & 90 n & 228 n \\
3 & $p_d$ & 202 n & 449 n \\
4 & $p_d^2$ & 424 n & 891 n \\
5 & $p_d^3$ & 866 n & 1774 n \\
\hline
\end{tabular}
\caption{Nombre d'opération par cellule des tests
pseudo-aléatoires de fautes de type k-coupling.
\label{table-compar}}
\end{center}
\end{table}
%%%% fin exemple %%%%

# 7.13 # Comment diviser une cellule par une diagonale ?
--------------------------------------------------------
Il faut utiliser le package 'slashbox' disponible par ftp à
ftp.tohoku.ac.jp dans
/pub/TeX/latex-styles/bear_collections/style-files/.

Exemple (de la doc) :
\begin{tabular}{|l||*{5}{c|}}\hline
\backslashbox{Room}{Date}
&\makebox[3em]{5/31}&\makebox[3em]{6/1}&\makebox[3em]{6/2}
&\makebox[3em]{6/3}&\makebox[3em]{6/4}\\\hline\hline
Meeting Room &&&&&\\\hline
Auditorium &&&&&\\\hline
Seminar Room &&&&&\\\hline
\end{tabular}

# 7.14 # Comment définir une colonne en mode mathématique dans
--------------------------------------------------------------
un tableau ?
------------
Avec le package 'array' disponible sur CTAN dans
/macros/latex/packages/tools/, il suffit de déclarer
\begin{tabular}{>{$}c<{$}cc} pour avoir une colonne en mode
mathématique et deux colonnes de texte.

# 7.15 # Comment modifier le nombre de tableaux par page ?
----------------------------------------------------------
cf. question 8.3.

# 7.16 # Comment définir un séparateur de colonne ?
---------------------------------------------------
@{symbole} entre deux descripteurs de colonnes permet de définir
"symbole" comme séparateur de colonnes. Cela remplace |.

Exemple :
\begin{tabular}{|l @{\textbf{ est }} l|}
\hline
Le lion & féroce. \\
Le chien & fidèle. \\
\hline
\end{tabular}
%%%% fin exemple %%%%

# 7.17 # Comment obtenir des lignes partielles dans un tableau ?
----------------------------------------------------------------
La commande \cline{ColonneDebut-ColonneFin} est faite pour ça.

Exemple :
\begin{center}
\begin{tabular}{|c|c|c||c|c|c|c|c|c|c||c|}
\hline \hline
a & b & c & d & e & f & g & h & i & j & k \\
\cline{1-4} \cline{6-6} \cline{8-9}
1 & 2 & 3 & 4 & 5 & 6 & 7 & 8 & 9 & 10 & 11 \\
\hline
\end{tabular}
\end{center}
%%%% fin exemple %%%%

# 7.18 # Comment éviter que du texte de grande taille atteigne
--------------------------------------------------------------
le cadre des cellules ?
-----------------------
* Il faut pour cela utiliser la commande \strut qui simule un
objet vertical invisible, après le changement de fonte.

Exemple :
\begin{tabular}{|l|}
\hline
{\large HELLO} dfg \\
\hline
\end{tabular}
\begin{tabular}{|l|}
\hline
{\large\strut HELLO} dfg \\
\hline
\end{tabular}
%%%% fin exemple %%%%

* Il existe l'équivalent mathématique : \mathstrut.

* On peut également agrandir la hauteur des lignes avec la
commande \arraystretch :
\renewcommand{\arraystretch}{1.5}

# 7.19 # Comment fixer la largeur d'un tableau ?
------------------------------------------------
* Le package 'tabularx' disponible sut CTAN dans
/macros/latex/packages/tools/ permet de définir une largeur de
tableau.

* Le package 'easytable' disponible sur CTAN dans
/macros/latex/contrib/supported/easy/ permet facilement de fixer
des largeur de colonnes ou de lignes.

# 7.20 # Comment tracer des traits discontinus ?
------------------------------------------------
Il faut utiliser les packages 'hvdashln' et 'array'.

Exemple :
dans le préambule du document :
\usepackage{hvdashln,array}
\setlength{\hdashlinewidth}{.5pt}
\setlength{\hdashlinegap}{2pt}

dans le texte :
\[
\left[ \begin{array}{ccc@{}>{\vdashline}c}
a_1 & b_1 & c_1 & d_1 \\
a_2 & b_2 & c_2 & d_2 \\
a_3 & b_3 & c_3 & d_3 \\
a_4 & b_4 & c_4 & d_4
\end{array} \right]
\]
%%%% fin exemple %%%%

# 7.21 # Comment fixer la taille et justifier une colonne ?
-----------------------------------------------------------
Le problème qui se pose lorsqu'on utilise :
\begin{tabular}{|>{\raggedright}p{0.33\textwidth}<{}|
>{\raggedleft}p{0.66\textwidth}<{}|}
A& \\
B&C \\
\end{tabular}
vient du fait que \\ est redéfini par tabular et les commandes
ragged. On ne peut donc plus utiliser \\ dans un texte.

Il existe différentes solution pour palier ce problème :
* on peut corriger ce problème en incluant les lignes suivantes
dans le préambule du document :
\makeatletter
\newtoks\fintableau
\let\fintableau\@arraycr
\makeatother

* avec le package 'array', il suffit d'utiliser
la commande \tabularnewline[<vertical space>] à la place de la
commande \\.

Exemple :
\begin{tabular}{|r|r|>{\raggedleft}m{190pt}|}
\hline
col1 & col 2 & col3 \tabularnewline
une & deux & trois \\ quatre \tabularnewline
\hline
\end{tabular}
%%%% fin exemple %%%%

* on peut également inclure la nouvelle commande suivante dans
le préambule du document :
\newcommand\PreserveBackSlash[1]{\let\temp=\\#1\let\\=\temp}
puis en utilisant :
\begin{tabular}{|r|r|>{\PreserveBackSlash\raggedleft}m{190pt}|}

# 7.22 # Comment faire une liste de tableaux ?
----------------------------------------------
Il suffit d'inclure la commande \listoftables à l'endroit où
l'on veut inclure cette liste. Sachant que cette liste fait
référence aux pages où apparaîssent les tableaux, il faut
enchaîner au moins deux compilations LaTeX pour que toutes les
références soient exactes.

--
mpk. _ _
/ |/ |
_ _ () () \
/o\/o\\\\\||\\/
\vvvv/////|\\\\
Bad Wally. wwww**** \\\\\\
---------------------------------------------------------------
Marie-Paule KLUTH Marie-Pa...@aar.alcatel-alsthom.fr
...............................................................
Alcatel Alsthom Recherche Tel : +33 (0)1 69 63 12 68
Route de Nozay Fax : +33 (0)1 69 63 18 12
91460 Marcoussis FRANCE
---------------------------------------------------------------

0 new messages